OB ExamMaster Questions

Réussis tes devoirs et examens dès maintenant avec Quizwiz!

A 30-year-old woman presents to the office complaining of an inability to achieve pregnancy after over a year of trying. What is the couple's least likely cause for infertility? A Azoospermia B Ovarian failure C Endometrial disease D Tubal disease E Pituitary disease

Pituitary disease E In the vast majority of women who are infertile or subfertile, the issue is due to either ovarian dysfunction or structural abnormalities, with hormonal abnormalities accounting for only 10%.

A 20-year-old woman is brought to the emergency room (ER) by the police. She was assaulted and raped 3 hours ago. Her last menstrual period (LMP) was 22 days ago, and her period lasts for 4 - 5 days with a 28-day menstrual cycle. She is not using any contraceptive method. What hormone is mainly produced by the ovary at this point of her menstrual cycle? Answer Choices 1 Estrogen 2 Follicle stimulating hormone (FSH) 3 Inhibins 4 Luteinizing hormone (LH) 5 Progesterone

progesterone Explanation The correct response is progesterone. The 5th day of the menstrual period corresponds to the follicular phase of the cycle, which starts from the first day of bleeding to the day of the LH peak. In this phase, the FSH, which is produced by the pituitary, stimulates the development of follicles in the ovaries, with only 1 follicle dominant at the end. During the second half of this phase, the granulosa cells of the follicle begin to produce estrogen. Ovulation occurs approximately at the middle of the menstrual cycle (day 14). At the end of the follicular phase, a rise in the level of estrogens takes place, preceding the LH peak. The ovulation will succeed by an average of 30 hours after this peak. The 22nd day of the menstrual period corresponds to the luteal phase of the menstrual cycle. The luteal phase starts from the day of ovulation to the first day of the menstrual period. In this phase, the ovarian corpus luteum begins to produce progesterone. If there is fecundation fertilization of the ovum, the production of progesterone continues for 4 to 5 weeks, until the placenta can take over and produce enough of the hormone to maintain pregnancy. If there is no fertilization, progesterone levels decrease and the menstruation occurs. Estrogens are produced by the granulosa cells of the follicle. They are very important in the follicular phase; their level is very low at the beginning of this phase and reaches a maximum point at the middle of the cycle, just before the LH peak. They are the feedback for the LH production and (in part) for the FSH production. Inhibins are also hormones produced by granulosa cells of the follicle, and they are part of the feedback for the FSH. Inhibin B levels rise during the luteal-follicular transition, are highest during the mid follicular phase, then go up again during the LH peak, and finally decrease in the late follicular phase. Inhibins A levels decrease during the late luteal phase.

A 28-year-old woman presents with abdominal pain and vaginal bleeding. The pain began last night as a dull ache in the right lower quadrant, but this morning it became much more severe. She also complains of dizziness and nausea. She cannot recall when her last menstrual period was, but she says that she began bleeding yesterday and has a light menstrual flow. On examination, the patient is afebrile; pulse is 100/min, BP is 86/60 mm Hg, and RR is 20/min. Physical exam reveals moderate to severe tenderness in the right lower quadrant with rebound. Pelvic exam reveals a small amount of blood at the cervical os with cervical motion tenderness. You order a variety of laboratory tests on this patient. Question What is the most likely diagnosis? Answer Choices 1 Acute appendicitis 2 Acute salpingitis 3 Tubo-ovarian abscess 4 Ruptured ectopic pregnancy 5 Ruptured ovarian cyst

ruptured ectopic pregnancy Explanation Ruptured ectopic pregnancy is probably the most life-threatening condition in gynecology. Any practitioner that cares for reproductive-aged women must keep a high index of suspicion for this dangerous event in order to avoid a delay in diagnosis that may be fatal. Ectopic pregnancy is defined as extra-uterine implantation of a pregnancy; it occurs most commonly in the fallopian tube (96%). Most tubal pregnancies are found in the distal 2/3 of the tube. 2% of ectopic pregnancies are uterine ectopic pregnancies (interstitial); other sites include the cervix, ovary, or pelvic and abdominal cavities, although these implantations are rare. The major risk factors for ectopic pregnancy are a prior tubal infection or tubal surgery, a prior ectopic pregnancy (giving a risk of 10-25+% for recurrence), a history of diethylstilbestrol (DES) exposure, or presence of an IUD (intrauterine device). Ectopic pregnancy occurs in 0.5 - 1.0% of pregnancies; however, the incidence of ectopic pregnancy is increasing in recent years. This increase is probably a reflection of the increasing incidence of pelvic inflammatory disease, increased use of IUDs, and increasing incidence of tubal ligation for sterilization and other tubal surgeries. A tubal pregnancy will usually present with a history of vaginal spotting and crampy pain as early as 4 to 5 weeks after the last menstrual period (LMP). The tubal pregnancy will usually bleed slowly at first, but when the pregnancy expands to the point of tubal rupture, patients can present with rapid hemorrhage, experiencing shock and then death. On physical exam, the patient will exhibit tenderness and often peritoneal irritation, especially if blood has leaked out of the fallopian tube. The uterus will be enlarged, but it will be smaller than expected for the age of the pregnancy. There is often cervical motion tenderness, and an adnexal mass may be felt. Abdominal pain with vaginal bleeding accompanied by hypotension, marked abdominal guarding, and rebound tenderness is suggestive of ruptured ectopic pregnancy. A serum pregnancy test may reveal a smaller quantitative value than expected by LMP. If the diagnosis is in question and the patient stable, serial beta-hCG values may be estimated. In ectopic pregnancy, the serial beta-hCG does not rise appropriately with respect to the gestational age (doubling every 48 - 72 hours). However, transvaginal ultrasound is usually utilized to look for the absence of an intrauterine gestational sac and the presence of an adnexal mass, which confirms the diagnosis of ectopic pregnancy. If a patient has a beta-hCG level of 1,500 mIU per mL or greater, but the transvaginal ultrasonography does not show the presence of an intrauterine gestational sac, then ectopic pregnancy should be suspected. Culdocentesis, with the finding of old, non-clotting blood, used to be performed commonly in the assessment of tubal pregnancy, but it has largely been replaced by ultrasound because this imaging technique has increased sensitivity. An untreated ectopic pregnancy is usually fatal. Early diagnosis is the key to appropriate treatment, and the mortality rate for ectopic pregnancy in the US is currently 10-15%. The treatment for ectopic pregnancy is usually surgical, especially if there has been tubal rupture. However, some early ectopic pregnancies may be treated with methotrexate. These patients must be followed carefully with serial serum pregnancy tests and ultrasounds, and up to 1/3 of these patients will still require eventual surgical intervention. While appendicitis, salpingitis, ovarian cyst, and tubo-ovarian abscess may also present with abdominal pain, the absence of specific symptoms for each of these helps in ruling them out; symptoms of abdominal pain, vaginal bleeding, and amenorrhea are characteristic for ectopic pregnancy.

A 22-year-old couple presents for the evaluation of the primary infertility. The woman has no symptoms; her family and gynecological history are not relevant. Her spouse regularly takes pancreatic enzymes, albuterol, and vitamins; he also frequently takes antibiotics because of the presence of the recurrent respiratory tract infections, bronchiectasis, and bronchiolectasis; there is also exocrine pancreatic insufficiency and intestinal dysfunction. Question What is the next step in the evaluation of their inability to conceive? Answer Choices 1 Sweat test 2 Serial ovarian sonography 3 Semen analysis 4 Progesterone test 5 Hysterosalpingogram

semen analysis Explanation The correct response is semen analysis. Infertility is due to the female partner 1/3 of the time, the male partner 1/3 of the time, and both partners or the unknown problems in the remaining 1/3 of cases. In this case, the husband has signs and symptoms that could cause problems conceiving. The most common causes of the male infertility are structural abnormalities, sperm production disorders, ejaculatory disturbances, and immunologic disorders. Symptoms, signs and the therapy this patient is is receiving should raise the possibility that he suffers the cystic fibrosis (CF). It is estimated that more than 95% of CF male patients are infertile. The most common findings is azoospermia due to the epididymal obstruction and/or the failure of the vas deferens to develop properly. A small minority of male CF patients are still fertile, and for that reason sperm analysis should be routinely offered in all CF patients. Sample semen is evaluated for volume, sperm count, motility and morphology. Patients with poor semen quality or numbers may be prepared for intrauterine insemination by washing and concentrating the ejaculate. Symptoms, signs, and the therapy that the husband is receiving should raise the possibility that he suffers the cystic fibrosis (CF). The diagnosis of CF rests on the combination of clinical criteria and abnormal CFTR (cystic fibrosis transmembrane conductance regulator), as documented by elevated sweat chlorde values, nasal potential difference responses, and the administration of the various solutions and by CFTR mutation analysis. Even if it is estimated that more than 95% of male CF patients are infertile, a minority with certain mutations is still fertile. For that reason, in all CF patients, sperm analysis should be routinely offered along with reproductive health education. Serial ovarian sonographic evaluations may help in demonstrating the existence of the ovulation: the development of a mature antral follicle and its subsequent collapse during ovulation. It is time consuming and is not always accurate; however, it may be helpful in supporting the diagnosis of polycystic ovary syndrome (PCOS). There is no reason to perform this test in a female patient with a non-contributiong personal and family history when the most probable cause is a husband's disease. The progesterone test is a test used to confirm ovulation; blood is drawn about 7 - 10 days after ovulation at a time when progesterone levels peak, and the level of progesterone helps in the confirmation of ovulation. There is no reason to perform this test in a female patient with a non-contributing personal and family history. Hysterosalpingogram (HSG) is a test of the patency of the Fallopian tubes and exploration of the atrial cavity. Dye is injected through the cervix and up into the uterus and Fallopian tube; its movement is observed on a video screen. There is no reason to perform this test in a female patient with a non-contributing personal and family history.

A 25-year-old female presents for an ultrasound after having a positive home pregnancy test. She has an unremarkable past medical history and physical exam. She states she has been feeling fine without any abdominal discomfort or vaginal bleeding noted. On ultrasound you determine she is 10 weeks pregnant. You note a noncomplex unilateral mass on her left ovary measuring 2 cm in diameter. Which additional history would support your suspected diagnosis? A History of herpes simplex virus B She used clomiphene to conceive C Two previous Caesarian sections D 20-pound unintentional weight loss

she used clomiphene to conceive B Patients who used assisted reproduction, such as clomiphene, present a special subgroup as their ovaries frequently have ovarian cysts. This is common during the first trimester due to ovarian hyperstimulation. Herpes (A), Cesarean sections (C), and miscarriages (E) are not proven to increase the risk of functional ovarian cysts. A 20-pound unintentional weight loss (D) would have you consider a malignant cause, which is much rarer than a functional ovarian cyst.

Twins are at a higher risk for growth problems and prematurity, although uncommon defects unique to twins occur. Which of the following is the most common developmental defect in a monochorionic twin pregnancy? Answer Choices 1 Conjoined twins 2 Acardiac twins 3 Twin-twin transfusion 4 Twin embolization syndrome 5 Monoamniotic, monochorionic twins

twin-to-twin transfusion Explanation Twin-twin transfusion syndrome occurs in approximately 10-20% of monochorionic twins. Twin-twin transfusion syndrome (TTTS) is a condition that affects monochorionic twin pregnancies, in which blood is shunted from one twin to the co-twin through vascular anastomoses in the placenta. The donor twin becomes progressively anemic, hypotensive, and hypovolemic with resulting oligohydramnios and lagging growth. The recipient twin becomes polycythemic, hypertensive, and hypervolemic with resulting polyhydramnios and normal growth. The prognosis is fatal if left untreated, mortality rates approaching 70-90% in such cases. The sonographic findings suggestive of TTTS includes monochorionicity, discrepancy in amniotic fluid in the amniotic sacs with polyhydramnios of one twin (largest vertical pocket greater than 8 cm) and oligohydramnios of the other (largest vertical pocket less than 2 cm), discrepancy in size of the umbilical cords, cardiac dysfunction in the polyhydramniotic twin, and significant growth discordance (often > 20%). An acardiac parabolic twin (parasitic twin that fails to develop a head, arms and heart) occurs in approximately 1:35,000 pregnancies. Conjoined twins occur in 1:50,000 pregnancies. Monoamniotic, monochorionic twining is when the twins share one sac with both the amnion and chorion being a single sac. Twin embolization syndrome (TES) is a complication associated with monozygotic twins, following in utero demise of the co-twin.

A women is being evaluated at her 36-week obstetrical appointment. She is not obese, her bladder is empty, and she does not have any complications. The fetus is in a cephalic position by Leopold maneuver. You measure her fundal height. What should it measure? A 32 to 34 cm B 32 to 36 cm C 34 to 38 cm D 35 to 39 cm E 32 to 39 cm

34 to 38 cm Fundal height in an uncomplicated, normal weight pregnancy should be within 1 to 2 cm per week of gestation in pregnancies above 20 weeks.

You are caring for a 29-year-old G1P0 at who is pregnant with twins. She has received routine obstetrical care and her pregnancy has been uneventful to date. What is the average gestation age for twins at delivery? A 33 to 34 B 39 to 40 C 35 to 36 D 36 to 37 E 37 to 38

36 to 37 D The average length of gestation for a single fetus is 40 weeks; the average age of gestation decreases with increasing number of fetuses.

A women presents to the labor department complaining of contraction every 3 to 4 minutes for the last 3 hours. She is a G1P0 at 40 weeks gestation. Her pregnancy is uncomplicated; her group B strep culture is negative. Physical exam vitals are normal, the baby is cephalic in a +2 station, and the bag of water is intact. The fetal heart monitor reveals fetal heart tones in the 140s with contractions every 3 minutes lasting 45 seconds. Her cervix is 4-cm dilated and 50% effaced. What is the expected rate of cervical dilation? A .5 cm per hour B 1 cm per hour C 1.2 cm per hour D 1.5 cm per hour E 1.7 cm per hour

1.2 cm per hour C This patient is in stage one active labor and is a primigravida. She should expect 1.2 cm of dilatation per hour; multigravida women can expect a faster rate of dilatation.

A 25-year-old woman brings in her menstrual calendar as part of a preconceptional counseling visit. Her cycles are regular, occurring every 30 days and lasting 3-4 days. She has mild cramping on days 1 and 2 that is easily relieved by ibuprofen or acetaminophen. On what day of her cycle is she most likely ovulating? A 12 B 14 C 16 D 18 E 20

16 C Day 1 of menses is the start of a new menstrual cycle. In normally menstruating women, the luteal phase is stable at 14 days, i.e., ovulation ordinarily occurs 14 days before the onset of the next menses. Therefore, in a woman with a very regular 30-day cycle it is most likely to occur on day 16. She may, however, become pregnant if she times intercourse for two days before (B) or after (D) the day of ovulation, due to other factors such as the duration of activity of sperm. Days 12 (A) and 20 (E) are at the margins of the period of fertility, but neither is the day of likely ovulation.

A 25-year-old woman brings in her menstrual calendar as part of a preconceptional counseling visit. Her cycles are regular, occurring every 30 days and lasting 3-4 days. She has mild cramping on days 1 and 2 that is easily relieved by ibuprofen or acetaminophen. On what day of her cycle is she most likely ovulating? A 12 B 14 C 16 D 18 E 20

16 Day 1 of menses is the start of a new menstrual cycle. In normally menstruating women, the luteal phase is stable at 14 days, i.e., ovulation ordinarily occurs 14 days before the onset of the next menses. Therefore, in a woman with a very regular 30-day cycle it is most likely to occur on day 16. She may, however, become pregnant if she times intercourse for two days before (B) or after (D) the day of ovulation, due to other factors such as the duration of activity of sperm. Days 12 (A) and 20 (E) are at the margins of the period of fertility, but neither is the day of likely ovulation.

You are caring for a pregnant woman who is Rh-negative. The father of the child is Rh-positive and heterozygous. What percent chance will the fetus have of being Rh-positive? A 0% B 25% C 50% D 75% E 100%

50% C The mother is negative and has no genetically positive material to pass to the child; the father is heterozygous, so only half of his genetics contain the positive antigens.

A G4P4 woman delivers a viable infant at 38 weeks gestation by normal spontaneous vaginal delivery. The infant has apgars of 7 and 8 (at 1 and 5 minutes respectively). What is the most crucial time for maternal and fetal physiologic changes to occur? A 30 minutes prior to birth B 60 minutes prior to birth C Birth D 30 minutes after birth E 60 minutes after birth

60 minutes after birth E The hour after birth is the most critical time for physiologic changes to occur, including maternal fluid shifts, hemorrhage, retained placenta, and fetal lung cardiovascular transitions.

A 19-year-old presents to clinic requesting emergency contraception. She is a G1P0Ab1 and a non smoker who has had intercourse and the condom broke. Her LMP was 3 weeks ago. Her PMH is negative. What would be the time frame for maximum efficacy for her to use emergency contraception? A 12 hours B 24 hours C 48 hours D 72 hours E 120 hours

72 hours Emergency contraception, both hormonal and IUD, reduces pregnancy rates for 120 hours, but there is a significant decrease in efficacy after 72 hours.

A 29-year-old G2 P1 who is term requests induction. Her pregnancy has been uncomplicated. She has been bothered by significant, poor quality contractions, which have caused her pain and interfered with her sleep. An elective induction is considered safe when the Bishops score is greater than what number? A 6 B 7 C 8 D 9 E 10

8 C A Bishop score greater than 9 is considered a positive predictor for safe delivery in a term pregnancy.

A 37-year-old woman, G3P2 at 30 weeks gestation, complains of lower extremity swelling and her weight is up 5 pounds this week. Her PMH is insignificant, and her other pregnancy was uncomplicated. Her BP baseline is now 142/92. On exam her BP is unchanged, her UA shows 2+ protein, and FHTs are 152. What is the cause of the protein in her urine? A Glomeruloendotheliosis B Glomerulonephritis C Renal vasospasm D Glomerular hemorrhage E Glomerular infarct

A Glomeruloendotheliosis The classic histological change that occurs in the renal system (in preeclampsia) is swelling and inflammation of the endothelium and of the glomeruli, which leads to endothelial leaking.

A 19-year-old G1 P0 presents to the emergency department complaining of abdominal pain. Her LMP was 2 months ago, and she has been spotting for the last two days. Her HCG is positive, and transvaginal ultrasound reveals no intrauterine gestational sac. The patient has an ectopic pregnancy, which has likely implanted in the fallopian tube. Why is the fallopian tube the most common non-uterine implantation site? A Lack of sub-mucosal layer in the fallopian tube B Inflammatory response in the endometrium C Proliferative endometrium D Extra myometrial tissue (fibroids) E Excessive trophoblastic proliferation

A Lack of sub-mucosal layer in the fallopian tube A The fallopian tube is the most common site of ectopic pregnancy, accounting for over 95% of ectopic pregnancies. The lack of a submucosal layer allows for easy wall access and implantation of the fertilized ovum. The increasing rate of Chlamydia infections in the U.S. also impacts the physical anatomy of the fallopian tube, and impacts where the ovum implants. Excessive trophoblastic activity does not impact where the ovum implants, and proliferative myometrium enhances uterine implantation if the fertilized ovum makes it to the uterine cavity.

A 34-year-old woman presents for her obstetrical checkup. She is a G2P1 and her pregnancy is uncomplicated to date. During her visit, what is the most important predictor of fetal well-being? A Normal maternal vitals and fetal activity B Maternal weight gain and fetal activity C Absence of contractions and fetal activity D Normal maternal vitals and absence of contractions E Maternal weight gain and normal maternal vitals

A Normal maternal vitals and fetal activity A Maternal vitals are a sign of maternal well-being; combined with normal fetal activity, this gives the best predictive value for fetal well-being without direct fetal surveillance.

A 17-year-old female presents to your office with intermittent menstrual pain. She is sexually active with a single male partner, consistently utilizing condoms. She reports that she has had this pain before, most commonly two weeks before her period, and that it has been increasing in severity over the past few months. You perform a pelvic examination and she has no lesions, discharge, or discomfort on bimanual and speculum exam. She is urinary chorionic gonadotropin (UCG) negative. Of the following, what is the most appropriate treatment for this adolescent? A Tell her she has mittelschmerz and prescribe an anti-inflammatory p.r.n. B Tell her she has primary dysmenorrhea and send her home on pain medications. C Recommend that she see a gynecologist to have an exploratory laparoscopic surgery to rule out endometriosis. D Refer her for a vaginal ultrasound to rule out uterine fibroids. E Draw a serum β-hCG.

A Tell her she has mittelschmerz and prescribe an anti-inflammatory p.r.n. Midcycle pain (mittelschmerz) is common in women with regular menstrual periods who are not taking birth control pills. These patients may commonly have midcycle spotting caused by an estrogen surge. There is no fever and no other abnormal bleeding such as that resulting from trauma to the cervix (e.g., coitus, douching). Pain usually occurs over several cycles. There is no history of intermittent lower abdominal pain. Examination at the time of mittelschmerz may reveal some lower quadrant tenderness with or without rebound. Bimanual examination may show localized tenderness. A palpable ovary may be present, but a history of regular menses, lack of fever, and negative pregnancy tests confirm the diagnosis. Mild analgesics, especially nonsteroidal anti-inflammatory drugs (NSAIDs), and reassurance are usually adequate for these patients.

During her active phase of labor, a provider has placed an external fetal monitor on a patient. The fetal heart rate (FHR) is noted to have a base line rate of 109, there are no late or variable decelerations, and the baseline variability is 0 beats per minute. What is this monitor tracing indicative of? A A normal FHR pattern B An indeterminate FHR pattern C An unreadable FHR pattern D Poor contact with monitor E A distress FHR pattern

A distress FHR pattern According to the Three-Tier Fetal Heart Interpretation System, recommended by the 2008 NICHD workshop on electronic fetal monitoring, the definition of fetal distress includes absence of baseline variability, and either bradycardia (FHR<110) or recurrent variable or late decelerations.

A 23-year-old primiparous woman visits her primary care provider for follow up with her pregnancy. Today her gestational age is 25 weeks. Her pregnancy has been uneventful so far. She has no personal or family history of diabetes mellitus. Her height is 5 feet 9 inches and her weight is 140 lbs. She has no chief complaints. The physician would like to screen for gestational diabetes and opts to perform a 50-g oral glucose challenge test. Regarding performance and interpretation of this test, what is true? Answer Choices 1 A standard fasting 50-g glucose challenge test with a glucose level of ≥100 mg/dL after 1 hour is abnormal and warrants further testing 2 A standard nonfasting 50-g glucose challenge test with a glucose level of ≥135 mg/dL after 1 hour is abnormal and warrants further testing 3 A standard fasting 50-g glucose challenge test with a glucose level of ≥110 mg/dL after 1 hour is abnormal and warrants further testing 4 A standard nonfasting 50-g glucose challenge test with a glucose level of ≥110 mg/dL after 1 hour is abnormal and warrants further testing 5 A standard fasting 50-g glucose challenge test with a glucose level of ≥95 mg/dL after 1 hour is abnormal and warrants further testing

A standard nonfasting 50-g glucose challenge test with a glucose level of ≥135 mg/dL after 1 hour is abnormal and warrants further testing Explanation Gestational diabetes affects approximately 5 - 6% of pregnancies in the United States. Most obstetricians adopt a policy of universal screening of pregnant women between 24 and 28 weeks gestational age using the oral 50 g glucose challenge test. The test is performed by administering a 50-g oral glucose load without fasting before the test. The test has a low specificity with handheld monitors; therefore venous blood should be used to obtain blood samples for glucose measurement. Most clinicians use a value above 135-140 mg/dL to prompt the need for follow-up testing (usually with 3-hour 100-g oral glucose testing, given after an overnight fast).

Your patient is a 16-year-old female who weighs 300lbs and is 5 ft. 2 inches tall. She has severe facial acne vulgaris, hirsutism, and amenorrhea. Her pediatrician has sent her to your 24-hour endocrinology clinical research station, because she thinks that the patient may have polycystic ovary syndrome. Since ovulation occurs after a sequence of steps involving changing blood hormonal levels, you could follow these levels in the patient during an entire ovarian cycle by an indwelling monitoring catheter. What would you look for to rule out the disease? Answer Choices 1 A very large rise in estrogen level during secretory phase 2 A very large drop in progesterone just before ovulation 3 A very large rise in luteinizing hormone (LH) just before ovulation 4 A very large rise in LH just before ovulation with the FSH level dropping during proliferative phase 5 A very large drop in estrogen level just before ovulation

A very large rise in luteinizing hormone (LH) just before ovulation Explanation There is a very large rise in LH level just before ovulation. FSH produces a maturing effect on the follicle and ova. In addition, the follicle produces a protein, inhibin that inhibits FSH production from the gonadotrope. This protein will become important in the events leading up to ovulation. When the follicle and ova are fully matured, LH binds to LH receptors and LH sponsors ovulation. But just prior to ovulation, the LH spike occurs by the following mechanism: at normal levels of the estradiol, the gonadotrope is inhibited, but if the levels of 17b-estradiol are very high, they produce a stimulatory effect on the gonadotrope. Just prior to ovulation, the levels of estradiol rise to extremely high levels. This stimulates the gonadotrope to produce both FSH and LH, but since FSH is inhibited by inhibin, it has a small peak, whereas LH levels rise dramatically. This is called the "LH spike." This dramatic rise in LH triggers ovulation. After ovulation, the follicle changes to become the corpus luteum. Under the primary control of LH, the corpus luteum synthesizes progesterone. Stimulation of the endometrial cells through these receptors leads to vascularization and thickening of the endometrium. These are the preliminary stages of readying the endometrial lining for trophoblast implantation. If fertilization of the ovum does not occur, then a series of events leads to the involution of the corpus luteum. Since the corpus luteum is dependent on LH, it eventually involutes under decreasing levels of LH. Involution of the corpus luteum leads to the decline of estradiol and progesterone levels. This, in turn, leads to the programmed cell death of the endometrial cells lining the uterine cavity and to the process of menses. The inhibition on the gonadotrope cell is released, and the cycle repeats itself.

A 17-year-old G1, P0 girl presents after being found in a crack house by local police. She was initially cooperative, but she is now experiencing severe abdominal pain, and she has developed vaginal bleeding. She says that she is pregnant, but she has not received any prenatal care. Examination reveals a blood pressure of 90/50 mm Hg, pulse of 120/min, and respiratory rate of 25/min. She is diaphoretic and clammy. Her uterus measures 25 cm from the pubic symphysis, but no fetal heart tones can be appreciated. There is a large amount of dark blood around the vagina. Ultrasound reveals an intrauterine fetal demise and a hyperechoic retroplacental hematoma. Labs are pending. Question What is the best course of action at this time? Answer Choices 1 Emergent labor by artificial rupture of membranes 2 Emergent labor induction with Pitocin 3 Administer fibrinogen 4 Emergency cesarean section 5 Administer crystalloids and obtain type and crossmatch for packed RBCs

Administer crystalloids and obtain type and crossmatch for packed RBCs Explanation This patient probably has a grade 3 placental abruption, characterized by external uterine bleeding and fetal demise, as well as maternal hemodynamic instability. Patients with placental abruption should have adequate intravenous access established. Crystalloid fluids are initially used to resuscitate the patient. Blood for transfusion should be typed and crossed (usually at least 4 units). Coagulation studies should be drawn to assess for signs of coagulopathy. Fibrinogen is usually given if the fibrinogen level is <100 mg/dL. Platelets may be administered if the platelet count is <50,000 /uL. Placental abruption (abruptio placenta) is defined as the premature separation from the uterus of a normally implanted placenta. As the placenta separates, there is a large amount of bleeding which irritates the uterus and causes uterine contractions, as well as fetal distress because fetal perfusion is compromised. The incidence of placental abruption ranges from 1 in 75 to 1 in 225 births. Placental abruption is classified by degree of separation. Grade 1 abruption presents with slight vaginal bleeding, mild uterine irritability, and a normal fetal heart rate tracing. Maternal blood pressure is normal, as is the maternal serum fibrinogen level. A grade 2 abruption is characterized by mild-to-moderate vaginal bleeding and pronounced uterine irritability, which may include tetanic uterine contractions. The patient may exhibit orthostatic blood pressure changes and often has an elevated pulse. The fetal heart rate usually shows some evidence of distress. Maternal fibrinogen levels are usually reduced to 150 to 250 mg/dL. A patient with a grade 3 abruption will have moderate-to-severe vaginal bleeding (unless concealed in the uterus), painful, tetanic uterine contractions, and hypotension. The fetus will be dead and the patient is likely to exhibit a coagulopathy with fibrinogen levels less than 150 mg/dL, thrombocytopenia, and reduced levels of clotting factors. Abruptio placenta is associated with several risk factors. Maternal hypertension is one of the most commonly associated risk factors identified in patients who develop placental abruption, especially those with grade 3 abruption (where 40 - 50% of patients will have hypertensive disease of pregnancy). A history of prior abruption, tobacco abuse, cocaine abuse, poor nutrition, and chorioamnionitis are also associated with an increased risk for abruption. Uterine trauma is also associated with a risk for placental abruption, causing 1 - 2% of grade 3 abruptions. Patients with trauma in pregnancy may present with minimal physical evidence of trauma, but they will still have a significant abruption that can progress from grade 1 to grade 3 within 24 hours. Multiple gestations and polyhydramnios can also cause placental abruption if the uterus decompresses rapidly during labor. Placental abruption will classically present with painful vaginal bleeding in the 3rd trimester of pregnancy (80% of patients present with bleeding), although the amount of bleeding may be concealed within the uterus (20%). Ultrasound is used to rule out placenta previa (the other common and dangerous cause of third trimester bleeding) and may identify placental abruption. Unfortunately, the ultrasound appearance of an abruption may lag behind the clinical degree of bleeding, so this imaging modality cannot rule out abruption. The prognosis and management of abruptio placenta depends greatly on the gestational age of the fetus, as well as the grade of the abruption. A grade 1 abruption with a term fetus can often be managed with a controlled induced delivery, and since blood is a strong uterine irritant, many of these patients will deliver relatively quickly. However, the management of preterm pregnancies is less clear and depends on the degree of abruption and fetal distress, with the risks of premature delivery measured against the risks of progression of the abruption.

A 24-year-old woman at 32 weeks of gestation presents with a fever, chills, generalized malaise, and vomiting. T=100.9oF, BP=110/70 mm Hg, P=100/bpm, RR 20/min On pelvic examination, she has acute fundal tenderness, the cervix is 2 to 3 cm dilated, she is 40% effaced, and vertex is at -1 station. Contractions are palpated and recorded every 5 to 10 minutes. Urinalysis shows no evidence of bacteria. On vaginal examination, membranes are ruptured. Question In addition to the administration of steroids, what is the most appropriate next step? Answer Choices 1 Administration of intravenous antibiotics and induction of labor 2 Assess fetal well-being witha non-stress test and biophysical profile 3 Fetal fibronectin testing 4 Performance of a cervical length ultrasound 5 Treatment with magnesium sulfate for tocolysis

Administration of intravenous antibiotics and induction of labor Explanation This patient has preterm premature rupture of membranes (PPROM) and she is in active labor. PPROM is defined as rupture of fetal membranes prior to 37 weeks' gestation. Given her high temperature and fundal tenderness, her clinical presentation is most consistent with chorioamnionitis. In the setting of ruptured membranes, the diagnosis is more likely. Management: Immediate antibiotics for chorioamnionitis therapy should be started, and the commencement of labor induction or augmentation should be considered once the patient is stabilized. Chorioamnionitis, labor, or non-reassuring fetal heart rate testing mandates delivery at any gestational age. In the absence of labor, chorioamnionitis, or non-reassuring fetal heart rate testing, patients with PPROM can be expectantly managed until 34 to 35 weeks' gestation with corticosteroids and broad-spectrum antibiotics: Corticosteroids: A complete course is indicated from 24 to 34 weeks' gestation. Because the degree of lung development is critical for premature neonates, betamethasone or dexamethasone are given to stimulate fetal lung development before delivery. This reduces the risk of respiratory distress syndrome (RDS) and intracranial hemorrhage (ICH). A recent review of several studies found significant benefits and no increase in risk of infection for mothers or infants given steroids. Current recommendations are that all patients with PPROM before 32 weeks receive steroids. Broad-spectrum antibiotics: A 7-day course is given. Antibiotics prolong the latency period and improve perinatal outcomes in patients with PPROM. Fetal well-being is assessed daily with a non-stress test, and a follow-up biophysical profile as needed is used for ongoing monitoring following pharmacologic intervention. Fetal fibronectin is a glycoprotein that plays a role in fetal membrane adhesion. It can be detected in the cervicovaginal fluid in the late 2nd and early 3rd trimester, and it has been associated with preterm birth. Fetal fibronectin testing may be useful in women with symptoms and negative tests because the negative predictive value is greater than 95%. This may avoid unnecessary treatment. However, the poor positive predictive value creates clinical ambiguity in patients who test positive. It is a diagnostic tool and would not be useful in this case. Tocolysis with magnesium sulfate is only useful for the immediate 48 hours following membrane rupture while achieving steroid therapy. Treatment beyond this period has not found to be effective; in fact, it may be harmful.

When is RhoGAM (D immunoglobulin) given to Rh-negative patients? Answer Choices 1 At the time pregnancy is diagnosed 2 At the beginning of the 2nd trimester 3 If the pregnancy continues beyond the estimated due date 4 After delivery if the infant is found to be Rh negative 5 After delivery if the infant is found to be Rh positive

After delivery if the infant is found to be Rh positive Explanation D immunoglobulin is administered to Rh negative mothers to prevent isoimmunization to the D antigen from an Rh positive fetus. It is recommended that all pregnant patients have a determination of blood type and antibody screening as part of their initial prenatal workup. A negative antibody screen indicates a non-isoimmunized mother who should have a repeat screen at 28 weeks and then have D immunoglobulin administered if her repeat screen remains negative. Once the D antibody is present, the patient is D-sensitized and there is no longer any benefit to the use of D immunoglobulin. Additional times in a pregnancy when the administration of D immunoglobulin to an Rh negative mother would be appropriate include: after spontaneous or induced abortion, after ectopic pregnancy, after any procedure which invades the uterine cavity (amniocentesis, chorionic villus sampling, fetal surgery), and after external cephalic version of a breech infant. Finally, D immunoglobulin should be administered post-partum to a patient whose infant is confirmed to be Rh positive. There is no indication to administer RhoGAM post-partum if the fetus is Rh-negative.

A 29-year-old G3P2 presents to the office for her obstetrical visit. She is currently 16 weeks gestation by ultrasound. Her pregnancy has had no complications to date. She is a non-smoker and takes her prenatal vitamin. What routine obstetric lab should be offered at this visit? A Amniocentesis B Alpha fetal protein screen C 3D ultrasound D D. Glucose challenge E E. HIV testing

Alpha fetal protein screen AFP testing is only available between 15 and 20 weeks gestation. HIV is done on initial visit and amniocentesis is offered for risk factors, advanced age, or abnormal AFP. 3D ultrasound is not routine standard of care, and diabetic screening is done between 24 and 32 weeks gestation.

A 37-year-old female presents to the labor and delivery department complaining of intermittent pain and contractions. Upon arrival, she also complains of vaginal bleeding. She is a G3P2 at 39 weeks gestation; no other prenatal complications are noted. She is a non-smoker. A physical exam reveals the following: P 90, BP 130/80, T 98.7°F, abdomen gravid, positive bowel sounds, and left lower quadrant tenderness noted. A sterile speculum exam reveals the cervix to be dilated 8, fetus is cephalic, and membranes are intact. The fetal monitor reveals heart tones in the 140s with mild, decreased variability and good quality contractions noted. Delivery is felt to be imminent, and vaginal delivery has been determined to be the best course of action. What will likely decrease bleeding and shorten time to delivery? A Increased activity level B Amniotomy C Oxytocin therapy D Epidural placement E IV sedation

Amniotomy If the fetus is mature and vaginal delivery (versus c-section) has been determined to be the best course of action, then amniotomy may diminished amnionic fluid volume. This might also allow for better spiral artery compression, and serve to both decrease bleeding from the implantation site and reduce entry of thromboplastin into the maternal circulation.

You are monitoring a 30-year-old G2P1 at 40 weeks gestation, who is in an active stage of labor and is 6-cm dilated. The fetal heart tracing has a baseline heart rate of 140, with 7 to 10 beats of variability. With the last five contractions you have noted late decelerations. What would be the next most appropriate course of action? A Close observation of FHR tracing B Assessment of dilatation C Augment contractions with oxytocin D Intravenous analgesic E Surgical intervention

Assessment of dilatation B The presence of recurrent late decelerations should raise the suspicion for fetal distress. Vaginal evaluation for change in dilatation or cord prolapse, and to assess the fetal response to stimulation, are the first steps in evaluating the need for intervention.

You are taking care of a 32-year-old G2P1 at 39 weeks gestation in active labor. Her pregnancy is complicated by gestation diabetes. The fetal head delivered, but the anterior shoulder did not deliver with gentle downward traction. What would be the next most appropriate action? A More forceful traction and fundal pressure B Call for assistance and McRoberts maneuver C Call for assistance and more forceful traction D Call for help and fundal pressure

B Call for assistance and McRoberts maneuver Shoulder dystocia is an obstetrical emergency, and help should always be summoned. The McRoberts maneuver increased the AP diameter, thus accommodating a large head; subrapubic pressure can help dislodge the anterior shoulder, but simple fundal pressure continues to impact it against the pelvic bone.

A 24-year-old female, with a history of type 2 diabetes, presents with the inability to conceive after 14 months of unprotected sexual intercourse with her husband. Her vital signs are unremarkable and you calculate a BMI of 31. Physical examination reveals acne vulgaris and hirsutism. Which of the following treatment options for her infertility would be the most effective considering your suspected diagnosis? A Medroxyprogesterone acetate B Clomiphene citrate C Metformin D Spironolactone E Mini-pill (progestin only)

B Clomiphene citrate Clomiphene citrate is highly effective as the first line treatment for infertility in PCOS. It can be accompanied with metformin, weight loss, exercise, and exogenous gonadotropins when clomiphene fails. PCOS in over half of patients is accompanied with obesity, abnormalities in insulin control, metabolic syndrome, and infertility. Medroxyprogesterone acetate (A) and the mini-pill (E) are used for endometrial protection and with oral contraceptive pills. Metformin (C) will help with her diabetes. Spironolactone (D) is a diuretic, which acts as a weak androgen receptor antagonist.

A 30-year-old woman delivers a viable 7 pound 6 ounce female infant by normal spontaneous vaginal delivery. The infant was delivered and held below the introitus. The infant was dried, stimulated, and apgars were assigned. The cord was then clamped and the infant was placed on the maternal abdomen. Which of the following is a complication that can arise from this? A Hypovolemia B Hyperbilirubinemia C Hypoglycemia D Hyperglycemia E Hypoxia

B Hyperbilirubinemia B In a normal delivery, after the infant is delivered through the introitus it should not be held below it; excessive fluids can be passed to the infant, resulting in increased hematocrit and hemoglobin, which will hemolyze and cause hyperbilirubinemia.

A 24-year-old delivers twins by cesarean section. The twins are monozygotic. The placenta was fused. What does this implies? A Single ovum and single chorion B Single ovum and double chorion C Double ovum and single chorion D Double ovum and double chorion E Triple chorion

B Identical twins indicate single ovum. If the placenta is fused or double it means there are two chorions, and that the trophoblast differentiation occurred before day 3.

A 24-year-old delivers twins by cesarean section. The twins are monozygotic. The placenta was fused. What does this implies? A Single ovum and single chorion B Single ovum and double chorion C Double ovum and single chorion D Double ovum and double chorion E Triple chorion

B Single ovum and double chorion B Identical twins indicate single ovum. If the placenta is fused or double it means there are two chorions, and that the trophoblast differentiation occurred before day 3.

Which bone is the most susceptible and most often fractured at birth? A Calcaneus B Clavicle C Femur D Humerus E Patella

B The clavicle is the most common bone broken during childbirth. It often is associated with shoulder dystocia, but clavicular fractures can occur in uncomplicated pregnancies. They are usually of the greenstick variety and heal without complications. Calcaneal and patellar fractures are highly unlikely to occur since they are not long bones which are much more vulnerable to fracture. Fractures of the humerus and femur are possible during childbirth, but generally only in traumatic births. Humerus and femur fractures are much less common than clavicular fractures

A 20-year-old woman presents to the medical office to start birth control pills. She is engaged to be married soon. She has never been sexually active. She states she feels healthy, and she denies any current complaints. A summary of her past medical history includes: Medications: None Allergies: Sulfa medications Surgical history: None Medical history: No known conditions. OB/GYN history: Menarche age 13. Regular monthly menses, with no menstrual complaints. Family history: Paternal grandfather has diabetes and hypertension. Maternal grandmother had a stroke. Maternal grandfather had prostate cancer. Father has hypertension and history of a heart attack. Social history: Patient works as a retail clerk part-time and is attending college. She currently lives with her parents. She denies the use of tobacco, alcohol, and recreational drugs. Question What component of the physical exam is the most important before prescribing birth control to this patient? Answer Choices 1 Bimanual pelvic exam 2 Blood pressure measurement 3 Breast exam 4 Cardiovascular exam 5 Pap smear with vaginal speculum exam

Blood pressure measurement Explanation It is most useful and important to obtain a blood pressure measurement prior to initiation of combined hormonal contraceptives. Routine screening, including physical exam and tests, is generally not recommended prior to initiating contraceptives. If the patient is found to have severe hypertension, combination hormonal contraceptives should be avoided. Likewise, if blood pressure measurements rise dramatically after using these contraceptives, they should be discontinued and alternate methods initiated. A bimanual pelvic exam has been somewhat controversial, with no compelling evidence to continue the practice in asymptomatic women. In fact, exams viewed as invasive or embarrassing (such as pelvic and breast exams) can be seen as a barrier for preventing unintended pregnancies; some women may avoid encounters to obtain contraceptives in order to avoid the exams. Breast exam is another routinely performed exam; it has limitations in the asymptomatic patient population for screening. It becomes much more useful in screening for breast cancer in older women, who are higher risk for breast cancer. A breast exam is not routinely recommended prior to initiating birth control. A cardiovascular exam is not recommended prior to initiating birth control. Even with some cardiovascular disease in this patient's family history, the cardiovascular exam is likely be normal and would not be useful in initiating contraceptives. The Pap smear (short for Papanicolaou) with vaginal speculum exam is primarily a test for cervical cancer and pre-cancerous dysplasia. Many years ago, it was thought that birth control increased risk of cervical cancer. That myth has been disproved. There is no compelling reason to obtain a Pap smear in order to initiate birth control. Routine cervical cancer screening guidelines should be followed, which would indicate that this patient should wait until the age of 21 years before beginning Pap testing. If the patient noted vaginal complaints or desired testing for sexually transmitted diseases, a speculum exam would be useful.

The patient is a 26-year-old woman who presents to her gynecologist's office with a 4-month history of the chief complaint of amenorrhea. She has had some breast tenderness, but denies nausea, vomiting, fatigue, and abdominal pain. She was sexually active until about 6 weeks ago when she broke up with her boyfriend, but states that they used condoms. Question Which of the following choices would best describe the appearance of her cervix during an internal examination if the patient is 10 weeks pregnant? Answer Choices 1 purulent discharge from cervical os 2 strawberry-like appearance to cervix 3 bluish appearance to cervix 4 cervical os slightly open 5 blood from cervical os

Bluish appearance to the cervix Explanation Bluish appearance to the cervix is the correct answer. While some pregnant women can have a normal appearance to their cervix, most will have a bluish appearance. In addition, bimanual palpation will reveal the cervix to be soft and flexible. The uterus also increases in size with gestational age and will fill the pelvis around 12 weeks. Strawberry-like appearance to the cervix is not the correct answer, as this appearance is more likely to be seen in patients who have some form of cervicitis. In particular, patients with cervicitis caused by trichomonas usually have purulent vaginal discharge and a friable cervix with punctate hemorrhages that give it a "strawberry-like" appearance. Cervical os slightly open is not the correct answer, as it is not the most likely appearance of the cervix, but could actually be seen in the cervix of a pregnant female. However, the os is usually only slightly open if the patient is experiencing a spontaneous abortion, has an incompetent cervix, or is in the early stages of labor. Purulent discharge from the cervical os is not the correct answer. If purulent discharge is seen coming from the cervical os, the likely diagnosis has something to do with an infection. The patient may have cervicitis or pelvic inflammatory disease. Blood from cervical os is an incorrect answer, as it is not typically indicative of a pregnant patient. Blood coming from the cervical os is more likely to be seen in a patient currently having their menses, a patient experiencing a spontaneous abortion, cervical malignancy, and during some stages of labor. If a female is known to be pregnant and has blood coming from the os, there are likely to be other symptoms associated with various disorders such as placental abnormalities.

A 25-year-old woman and her husband have been using condoms and spermicidal foam for the 8 months since the birth of their baby. She plans to wean the baby from the breast sometime between a year and 18 months of age, but would like to begin a "less messy" method of contraception. Prior to her pregnancy she took combination oral contraceptives for several years without any difficulties. Which of the following is an appropriate recommendation for this woman? A "Breast feeding alone will prevent pregnancy until the baby is weaned." B "Condoms and spermicide are your only option until you wean the baby." C "Progestin-only pills are recommended for women who are breast feeding." D "A vaginal ring will not affect your milk production." E "You can start back on the same oral contraceptive you took before."

C "Progestin-only pills are recommended for women who are breast feeding." C The progestin-only pill is ideal for breast feeding mothers because this pill does not interfere with lactation the way combination pills do. Breast feeding (A) is reasonably effective in preventing pregnancy only as long as breast milk is the infant's only source of nutrition. Condoms and spermicide (B) are options, but not the only ones for breastfeeding couples. A vaginal ring (D) does decrease the amount of milk production somewhat, but may be an effective option if lactation is well established. Combination oral contraceptives (E) are not recommended for breastfeeding women.

A 30-year-old female G2 P2, who delivered via normal spontaneous vaginal delivery, presents complaining of increasing vaginal pressure, low back pain, and stress incontinence. What is the mostly likely cause of her condition? A Damage to the levator muscles B Increased intra abdominal pressure C Widening of the levator gap D Widening of the AP pelvis diameter E Endopelvic fascia remodeling and cervical elongation

C Widening of the levator gap Damage to the levator and increased abdominal pressure are known risk factors, but the widening of the gap with the associated risk factors is what allows the defect to occur. Fascia remodeling can occur instead of a prolapse defect.

Which of the following tests should an HIV-positive pregnant woman undergo in each trimester of pregnancy? A CD4+ lymphocyte count B cytomegalovirus serology C postpartum depression with controls D shielded chest radiography E venereal disease research laboratory (VDRL)

CD4+ count lymphocytic count A HIV-positive pregnant women should undergo CD4+ serology each trimester. Early in the pregnancy, they should undergo shielded chest radiography, CMV baseline testing, and tuberculosis testing with controls. Syphilis testing should be completed initially and as usually recommended later in pregnancy.

A young pregnant woman comes to her physician for a routine check-up. She tells her physician that she would like to test for any fetal abnormalities. After you discuss with her the tests available, she agrees on the procedure that allows for the earliest prenatal diagnosis in pregnancy. Which of the following test is the most likely? Answer Choices 1 Amniocentesis 2 Fetal Biopsy 3 Chorionic villus samplings 4 Umbilical blood sampling 5 Ultrasound

CVS Explanation Chorionic villus sampling can be performed as early as the eighth week of pregnancy under the guidance of an ultrasound. Chromosomal and biochemical abnormalities can be detected. This procedure has potential complications such as bleeding from the biopsy site, compromise of fetal membranes, and infection. Amniocentesis may be performed from 12-16 weeks of gestation. Penetration of the placenta is an unwanted complication. Amniocentesis should be considered if the woman is over 35-years-old, or if there is a previous child with a chromosomal anomaly.

A 26-year-old woman at 32 weeks gestation is brought via ambulance to the ER with a 30-minute history of convulsions; they occurred while she was at work. Her vital signs include a blood pressure of 160/95 mm Hg, heart rate of 84 beats/min, and respiratory rate of 22 breaths/min. On physical examination, she is noted to have lower extremity edema and is hyperreflexic. She was treated with intravenous magnesium sulfate at a rate of 2 g/hour for 12 hours, and she is now is hyporeflexic; her respirations are decreased. What would you prescribe this patient? Answer Choices 1 Diazepam 2 Oxygen 3 Continue magnesium 4 Calcium 5 Potassium

Calcium Explanation The clinical picture is suggestive of eclampsia. In this instance, she became hyporeflexic and her respirations decreased, indicating possible magnesium toxicity. This can be reversed by giving calcium gluconate. Diazepam does not reverse the effects of magnesium toxicity. Oxygen will also not reverse the effects of magnesium toxicity. Continuing the magnesium would worsen the condition. Potassium does not reverse the effects of magnesium toxicity.

You are caring for a 29-year-old G3P2 at 39 weeks gestation, who has been laboring for 6 hours. She is a diet-controlled diabetic. Her last child was 9 pounds 8 ounces. She has been completely dilated for 2 hours, and the fetal head is at a plus 2 station, which is unchanged. What is the next most appropriate course of action? A Begin oxytocin B Vacuum extraction C High forcep extraction D High forcep rotation E Cesarean section

Cesarian Section The patient is a diabetic with a history of a macrosomic infant; the likelihood of macrosomia in this infant is significant. Instrument delivery is not recommended if macrosomia is suspected. By definition, she has had an arrest of descent of the fetal head and one should be highly suspicious for macrosomia, in which case a c-section is the preferred method of delivery.

A 25-year-old primigravida presents at 8 weeks gestational age for her first prenatal visit. She denies any abnormal vaginal discharge or pelvic pain. On pelvic exam, you note cyanosis of the proximal vagina and cervix. Question What is this finding consistent with? Answer Choices 1 Acute cervicitis 2 Chronic cervicitis 3 Hegar's sign 4 Chadwick's sign 5 Cervical ectopic pregnancy

Chadwick's sign

A 40-year-old female is status post a dilatation and curettage for hydatidiform mole. On week 3 post surgery, her follow-up quantitative hCG level has elevated slightly. What is the most likely diagnosis? A Adenocarcinoma of the ovary B Adenocarcinoma of the uterus C Retained hydatidiform mole D Choriocarcinoma E Corpus luteum cys

Choriocarcinoma 5% of hydatidiform mole progress to choriocarcinoma; the longer the mole in intrauterine the higher the risk. Pre-surgical evaluation for mole removal includes a chest x-ray to rule out distant metastasis. hCg that either plateuas or elevates is choriocarcinoma until proven otherwise, and requires prompt evaluation.

our patient, a 48-year-old woman, presents with vaginal bleeding and states that she is "alarmed" because she is quite sure she is 2 months pregnant. History includes unremarkable live birth of a male child 7 years ago and a molar pregnancy a year ago. Examination reveals a uterus that is inappropriately large for gestational length and hCG levels are higher than expected. Fetal parts and heart sounds are not present. Your diagnosis is that of carcinoma, but you are able to reassure your patient that this neoplasm is of the type that is most sensitive to chemotherapy. What is your diagnosis? Answer Choices 1 Endometrioid carcinoma 2 Ovarian dysgerminoma 3 Choriocarcinoma 4 Serous cystadenocarcinoma 5 Ovarian teratoma

Choriocarcinoma Explanation Gestational Trophoblastic Diseases are a group of related diseases forming a spectrum from Benign Hydatidiform Mole, to Invasive Mole, to Placental-Site Trophoblastoma, and finally to Choriocarcinoma. Treatment of women who have nonmetastatic gestational trophoblastic disease is almost 100% successful, and allows reproductive function to be preserved. The cure rate for metastatic disease is approximately 90 percent. Serous cystadenocarcinoma occurs in the ovary and is a cystic or semi-cystic neoplasm. It usually occurs bilaterally An ovarian dysgerminoma is a malignant ovarian neoplasm, hypothesized to be derived from primordial germ cells of the sexually undifferentiated embryonic gonad. Endometrioid carcinoma resembles the typical carcinoma of the endometrium, as its name suggests, but is an ovarian carcinoma. This neoplasm may be seen with endometrial carcinoma. Ovarian teratoma is composed of tissues that are derived from three germinal layers, the endoderm, mesoderm, and ectoderm. Many teratomas will contain hair or teeth. These true neoplasms will usually present with lower quadrant pain.

A 40-year-old female is status post a dilatation and curettage for hydatidiform mole. On week 3 post surgery, her follow-up quantitative hCG level has elevated slightly. What is the most likely diagnosis? A Adenocarcinoma of the ovary B Adenocarcinoma of the uterus C Retained hydatidiform mole D Choriocarcinoma E Corpus luteum cyst

Choriocarcinoma 5% of hydatidiform mole progress to choriocarcinoma; the longer the mole in intrauterine the higher the risk. Pre-surgical evaluation for mole removal includes a chest x-ray to rule out distant metastasis. hCg that either plateuas or elevates is choriocarcinoma until proven otherwise, and requires prompt evaluation.

A 24-year-old female, with a history of type 2 diabetes, presents with the inability to conceive after 14 months of unprotected sexual intercourse with her husband. Her vital signs are unremarkable and you calculate a BMI of 31. Physical examination reveals acne vulgaris and hirsutism. Which of the following treatment options for her infertility would be the most effective considering your suspected diagnosis? A Medroxyprogesterone acetate B Clomiphene citrate C Metformin D Spironolactone E Mini-pill (progestin only)

Clomiphene citrate B Clomiphene citrate is highly effective as the first line treatment for infertility in PCOS. It can be accompanied with metformin, weight loss, exercise, and exogenous gonadotropins when clomiphene fails. PCOS in over half of patients is accompanied with obesity, abnormalities in insulin control, metabolic syndrome, and infertility. Medroxyprogesterone acetate (A) and the mini-pill (E) are used for endometrial protection and with oral contraceptive pills. Metformin (C) will help with her diabetes. Spironolactone (D) is a diuretic, which acts as a weak androgen receptor antagonist.

A 36-year-old gravida 5, para 3013 at 34 weeks gestational age presents to the Triage Unit of Labor and Delivery with a 2-hour history of painless bright red bleeding per vagina. She states that after urinating, she found the toilet "filled with blood" when she stood up. There is normal fetal movement. She had 1 Caesarean section for uncertain fetal status 2 years ago. Her other 2 term pregnancies resulted in normal vaginal deliveries. She denies recent intercourse, prior history of vaginal bleeding during this pregnancy, syncope, dizziness, and headache. Vital signs: BP is 100/72 mm Hg, and maternal pulse is 110 bpm; respirations are 18/min. The patient is afebrile. Fetal heart rate is160 bpm, absent variability, absent accelerations, and occasional variable decelerations; therefore it is a category II fetal heart rate tracing. Occasional irregular contractions are seen on tocodynamometer. Question What management step should be taken immediately? Answer Choices 1 Caesarean section 2 Complete blood count, type and hold, administration of IV fluids 3 Biophysical profile 4 Amniocentesis for lecithin and sphingomyelin 5 Induction of labor; anticipate normal vaginal delivery

Complete blood count, type and hold, administration of IV fluids Explanation The most likely diagnosis for this patient with 3rd trimester bleeding is placenta praevia. Obstetrical hemorrhage is still one of the most common etiologies of maternal death in the United States. In a patient with significant blood loss, attention must first be paid to determination of the degree of blood loss, volume expansion with isotonic IV fluids, and access to blood replacement if necessary. Caesarean section is not indicated at this time, as maternal complete blood count is not yet available, and as at this point the fetal heart rate tracing is Category 2. A biophysical profile may be of benefit in determining fetal status, but is not the most critical next step. Sonography to confirm placenta praevia is more critical than the biophysical profile at this time. Amniocentesis for lecithin and sphingomyelin may be of benefit in determining whether the fetus has achieved lung maturity, but it is not the next best step in managing this emergent situation. Induction of labor and normal vaginal delivery are contraindicated in placenta praevia.

A 20-year-old woman who was 3 months pregnant aborted a mass which resembled a bunch of grapes. The specimen that she passed is shown in Figure G3.5. Karyotype analysis of the specimen revealed that it was 46, XX. The MOST likely diagnosis is which of the following? Answer Choices 1 Incomplete abortion, normal placenta 2 Choriocarcinoma 3 Ectopic pregnancy 4 Partial (incomplete) hydatidiform mole 5 Complete hydatidiform mole

Complete hydatidiform mole Explanation A hydatidiform mole occurs once in about 2000 pregnancies. Its incidence is higher among women at the two extremes of maternal age than in other women. The serum and urinary chorionic gonadotropin levels are usually abnormally high. In complete mole, the cause is fertilization of an ovum which has lost its chromosomes. Almost all have a 46XX diploid pattern from the sperm by androgenesis. There are no fetal parts. Almost every chorionic villus is cystic and avascular. There is diffuse trophoblastic proliferation. About 2 percent develop choriocarcinoma. Partial mole is due to fertilization of an ovum by 2 sperms. It has a triploid 69XXY chromosomal pattern. Only some villi are cystic, and there is slight trophoblastic proliferation. Fetal parts may be present. Choriocarcinoma rarely develops from a partial mole. The first manifestation of the presence of a mole is often the excessively rapid enlargement of the uterus due to growth of the placental mass. This is usually followed, at about the third to fifth month of pregnancy, by uterine bleeding or abortion. Placental tissue passed at this time show that the villi are cystic and grape-like. Microscopically, the mole shows cystic avascular villi together with irregular clumps of large syncytial and cytotrophoblastic cells. Hydatidiform mole may be complete or partial. It is characterized by cystic chorionic villi. Choriocarcinoma may develop in a mole, especially the complete type. About 50 percent of gestational choriocarcinomas arise in moles. Many of these have been treated successfully with chemotherapy. Choriocarcinoma is a rare neoplasm. In about half the cases, it arises in women with a hydatid mole. The rest of the cases are about equally divided between abortions and full-term pregnancy. A few develop as a teratoma unrelated to a pregnancy. Grossly, the uterus containing a choriocarcinoma shows shaggy, fleshy hemorrhagic masses filling the uterine cavity and extending into the myometrium. Microscopically, the neoplasm is composed of clusters of large, bizarre, often multinucleated giant trophoblastic cells, extending into the myometrium and often filling the vascular sinuses. Necrosis and hemorrhage are seen, both in the tumor masses and in the myometrium. Very rarely both the primary neoplasm and the metastases have disappeared spontaneously. Numerous cures have been achieved by methotrexate and actinomycin D in choriocarcinomas related to a mole or pregnancy. Typically the tumor spreads locally to the vagina and parametrium, invades the veins, and metastasizes to the lungs, liver, and brain. Usually the neoplastic cells produce chorionic gonadotropin in both the primary growth and the metastases, sometimes with extremely high titers. Most ectopic pregnancies occur in a uterine tube. Any pathologic change which causes narrowing or blockage of the tubal lumen predisposes to tubal pregnancy. The narrowing may be intrinsic due to inflammation or extrinsic due to formation of adhesions around the tube. Tubal ligation is designed to prevent fertilization of ovum by sperm and thus any pregnancy. Tubal pregnancy is the most common form of ectopic pregnancy. It occurs about once in 200 pregnancies. The burrowing trophoblast usually erodes and ruptures the tube early in pregnancy. Sometimes tubal abortion occurs, the fetus being extruded from the lateral end of the tube into the abdominal cavity. Interstitial pregnancy is implantation in a cornu of the uterus in the interstitial portion of the uterine tube. 'Abdominal' pregnancy (peritoneal pregnancy) may result either from direct implantation of the fertilized ovum upon the peritoneum or from extrusion of the fetus of a tubal pregnancy into the abdominal cavity. Ovarian pregnancy is extremely rare and presumably results from fertilization of the ovum at the time of ovulation.

A 27-year-old G3P2 at 39 weeks gestation has been in labor for 6 hours. Her membranes ruptured 3 hours ago and revealed a large amount of clear fluid. She has previously delivered a 7 pound 2 ounce infant. She is having good quality contractions, and has been completely dilated and pushing for 55 minutes. The fetal head is +4, FHT 128. What would be the most appropriate management at this time? A Continue pushing B Instrument delivery C Emergency caesarean section D Non emergent caesarean section E Epidural pain control

Continue pushing A This is a multigravida patient with an adequate pelvis. She is making steady progress and there are no signs of maternal or fetal distress, so continued labor management is expected.

A 35-year-old female presents to discuss non-permanent methods of birth control. She is a G3P2, PMH negative. She is a smoker. Which method of contraception would have the lowest risk profile for her? A Combination low dose pill daily B Injectable progestin monthly C Transdermal combination patch D Levonorgestrel intrauterine device E Copper intrauterine device

Copper IUD E A women who is over 35 and smokes is at high risk for cardiovascular complications. All choices except the copper IUD contain hormones, which may increase the risk of complications. The copper IUD is long term but non-permanent.

A 24-year-old female, with a history of type 2 diabetes, presents with the inability to conceive after 14 months of unprotected sexual intercourse with her husband. Vital signs are as follows: Temperature 37˚ C, BP 115/85, P 65, RR 20. Her BMI is 31. Which physical examination finding would you expect to observe with your suspected diagnosis? A Undeveloped breasts B Palpable uterus on abdominal exam C Acromegaly D Coarse dark hair on the face, chest, and back E Egophony on lung exam

D Coarse dark hair on the face, chest, and back Polycystic ovarian syndrome (PCOS) affects 5-10% of reproductive age women. It is associated with hirsutism (50% of cases) (D), obesity (80% of cases), and virilization (20%), and those affected have an increased risk of diabetes mellitus, cardiovascular disease, and metabolic syndrome. PCOS patients are often infertile. A palpable ovary may be noted on thin individuals with enlarged ovaries, not the uterus (B). Undeveloped breasts are not a diagnostic indicator for PCOS (A) and acromegaly (C) is seen with Marfan's Syndrome. Egophony (E) is when there is consolidation of the lung on exam.

An 18-year-old G1P0 presents for evaluation of her amenorrhea. Her LMP was 2 months ago. Her HCG is positive, and she has been having vaginal spotting x 2 weeks. Her ultrasound reveals an intrauterine heterogeneous echogenic mass, without fetus or placenta. What is the most likely diagnosis? A Threatened abortion B Missed abortion C Ectopic pregnancy D Hydatidiform mole E Early intrauterine pregnancy

D Hydatidiform mole D The classic signs for a mole are a heterogenous mass without the placenta or fetus, and vaginal spotting is present all of the time. The presence of the mass intrauterine without fetus or placenta rules out ectopic and IUP, and therefore threatened abortion. Missed abortion is not echogenic.

A woman presents for a routine post-partum checkup four weeks after delivery of her child. She is currently breast feeding without difficulty. She tells you that she has been feeling anxious and very warm, despite the change of season into winter. She is happy as a mother and has not had problems caring for her baby. What is the next best step in the investigation of her symptoms? A Radioactive iodine uptake test B Serum total T4 level C Thyroid fine needle aspiration D Serum TSH level E MRI of the anterior pituitary

D Serum TSH level The correct choice is D, serum TSH level. This patient is presenting with post-partum thyroiditis. Thyroid dysfunction occurs in 2 to 5% of women after giving birth. It can recur with subsequent pregnancies and develop into long-term thyroid disease as well. Typically, women with this disorder first develop signs and symptoms of hyperthyroidism, which then later changes to hypothyroidism. Most symptoms resolve spontaneously within a few months. In the hyperthyroid stage of this disorder, blood tests will reveal a suppressed TSH level with elevated serum thyroid hormone levels, as is common in all forms of primary hyperthyroidism. Choice A, a radioactive iodine uptake test, would reveal little or no uptake, but this test should not be ordered in a woman who is breastfeeding. For choice B, a serum total T 4 level can be elevated, but is not as sensitive or specific as TSH as a screening test. Any protein status changes in the woman can cause an elevated total T 4 without any thyroid dysfunction. Choice C, a thyroid fine needle aspiration, can be performed, but is not likely necessary and would not be the best next step for this patient. Choice E, an MRI of the anterior pituitary, would only be suggested if there is a suspicion of a pituitary tumor. This is a rare cause of hyperthyroidism and would not be the best next step.

A 26-year-old woman is in the 24th week of her first pregnancy. She is in fairly good shape, and the pregnancy is progressing well. However, a fasting blood glucose done in the office shows it to be 146 mg/dl. It is repeated the next day, and the value is 142 mg/dl. An oral glucose tolerance test is ordered, which comes back as abnormal. She is diagnosed with gestational diabetes and advised to meet with her obstetrician. What should be the recommended regimen for this patient? Answer Choices 1 Diet and exercise until delivery 2 Diet and metformin if blood glucose remains high despite diet control 3 Diet and sulfonylurea if blood glucose remains high despite diet control 4 Diet, exercise, and insulin therapy if blood glucose remains high despite diet control 5 Diet and acarbose if blood glucose remains high despite diet control

Diet, exercise, and insulin therapy if blood glucose remains high despite diet control Explanation Gestational diabetes mellitus (GDM) should be treated early and optimally to reduce risks to the fetus. In pregnancy there is an increased resistance to insulin which causes high levels of glucose, lipids, and insulin. Placental lactogen and high levels of estrogen and progesterone cause this scenario. Risk factors for GDM include family history of diabetes, prior delivery of large baby, age >25 years, obesity, steroid use, glycosuria in first antenatal visit, ethnicity like Hispanic, African, Native American, Asian or Pacific Islander, polycystic ovarian disease (PCOS), and prior fetal loss of unknown cause. Hyperglycemia is harmful to the mother and the fetus and can cause increased perinatal mortality. Preeclampsia and polyhydramnios have a higher incidence in GDM. Trauma during birth and operative delivery are some other adverse outcomes. Macrosomia, or larger than normal fetus, is common. Metabolic abnormalities in the baby post delivery like hypoglycemia and hyperbilirubinemia, childhood obesity, attention deficit disorder (ADHD), and early diabetes may occur in the child. Universal screening of all pregnant women is recommended (though very low risk females may be excluded), usually at 24-28 weeks of pregnancy except if risk factors are evident like obesity, prior history of GDM etc, in which case screening should be done as early as possible. A random blood sugar of >200 mg/dL or fasting glucose of >126 mg/dL on 2 occasions is diagnostic of diabetes and does not require screening. Screening is done with a 50 gm oral glucose challenge test. A value of >140 mg/dL is positive, and a 3-hour glucose tolerance test is then done for definitive diagnosis. GDM is confirmed if 2 or more of the following are present: fasting glucose> 95 mg/dL, 1-hour glucose >180 mg/dL, 2-hour glucose >155 mg/dL, and 3-hour glucose >140 mg/dL. Referral to a nutritionist for strict diet control is a must to maintain euglycemia, prevent ketosis, and monitor adequate weight gain. Blood glucose should be measured before breakfast and 1 hour after each meal by the patient and moderate exercise is encouraged. Fasting blood glucose should be <90 mg/dL and post-prandial should be <120 mg/dL. If blood glucose remains high in spite of these measures, then insulin is the treatment of choice. Oral hypoglycemic agents are not approved for use in pregnancy in USA due to transplacental passage and fetal morbidity. If fasting blood glucose is high, then NPH insulin is used at bedtime in the dose of 0.2 units/kg body weight. If post-prandial glucose is high, then regular insulin is used before meals. If both are high, then a regimen of 4 injections a day needs to be started, including NPH before breakfast and at bedtime and regular insulin before each meal.

Which of the following, is a classical hydatidiform mole? Answer Choices 1 Diploid karyotype, absence of an embryo, swelling of all villi 2 Triploid karyotype, trophoblastic hypoplasia, viable embryo 3 Trophoblastic hyperplasia, normal karyotype, twin embryos 4 Vascular malformations, intraplacental mass lesions, fetal hydrops 5 Decreased uteroplacental blood flow, multiple placental lobes, vesicular pattern

Diploid karyotype, absence of an embryo, swelling of all villi

A 28-year-old pregnant woman at 18-weeks gestation presents because she has been exposed to fifth disease. The patient is currently asymptomatic. What can you tell her about human parvovirus B19 and pregnancy? Answer Choices 1 Less than 25% of adults are immune to this virus, and she should be treated with immunoglobulin 2 Parvovirus B19 has not been associated with any fetal effects 3 If she develops an infection, she will need to be followed with serial fetal ultrasounds 4 The virus is spread by fecal-oral transmission, and she should wash her hands very carefully 5 The risk of fetal loss in an infected mother is between 45-55%

Explanation Fifth disease is caused by human parvovirus B19, which is a DNA virus. Fifth disease, also called erythema infectiosum, is usually a mild exanthem of childhood, but infection of a pregnant woman can have severe fetal complications Fetal (transplacental) infection with parvovirus B19 can result in a variety of fetal complications, including fetal loss, especially if the infection is between gestational weeks 10 and 20. A common complication includes fetal hydrops; it is caused by damage to fetal hematopoietic tissue, and it causes severe anemia and a resultant congestive heart failure. The virus can also cause a fetal viral myocarditis, which further worsens cardiac function and fetal hydrops. More rarely, first trimester infections with parvovirus B19 can cause teratogenic effects, including multiorgan abnormalities. Between 30-60% of adults are immune to parvovirus B19, as evidenced by the presence of IgG to B19 in their serum; therefore, most pregnant patients are probably immune to this virus. However, if a pregnant patient develops a rash or aplastic crisis that may be consistent with fifth disease, then igG and IgM serologies for parvovirus should be drawn to evaluate for acute infection. If parvovirus B19 infection is diagnosed in a pregnant patient, then her physician may choose to follow her with serial fetal ultrasounds to evaluate for the development of fetal hydrops. In some cases, fetal umbilical cordocentesis has been used to detect fetal infection. In children, fifth disease is characterized by a classic "slapped-cheek" facial erythema; it is associated with fever and often GI or other systemic symptoms. Adult patients will demonstrate a rash. The rash may be reticular, morbilliform, or even purpuric. Adult patients often have fever, lymphadenopathy, and/or arthritis. Parvovirus B19 is also associated with an acute transient aplastic crisis. Unfortunately, fifth disease is infectious for days before the onset of the rash; thus, many obstetrical patients may be exposed, particularly if they work closely with children. The virus is spread by aerosolized respiratory droplets, and it has an incubation period of 4 to 14 days.

A 32-year-old healthy woman is asking for advice about contraception. She wants contraception that is both effective and convenient in terms of application. She thinks that oral contraceptive pills will work the best, and she is interested in additional benefits that the pill can provide (particularly cancer protection). Question You inform her that, in her case, oral contraceptives may reduce the risk of death from what condition? Answer Choices 1 Breast cancer 2 Ovarian cancer 3 Colon cancer 4 Cervical cancer 5 Hepatic cancer

Explanation Ovarian cancers are often diagnosed in the advanced stages; this is the reason why ovarian cancer accounts for more deaths than any other gynecological cancer. The risk of ovarian cancer is reduced by 30% with pill use for <4 years, by 60% with use for 5 - 11 years, and by 80% after 12 or more years of use. Therefore, protection against ovarian cancer reduces the risk of gynecological cancer death. The relationship between breast cancer and OCP is controversial. Some research finds no increased risk of breast cancer in current or former users of OCPs aged 35 - 64, in women with a family history of breast cancer, and in women who started oral contraceptive use at a young age. Other research has found a very low risk of breast cancer in women of childbearing age (relative risk of 1.1 - 1.2, depending on other variables). This small increase is not substantially affected by duration of use, dose, type of component, age at 1st use, or parity. Even 10 years after the discontinuation of oral contraceptive use, no difference in breast cancer incidence between past users and those who have never used oral contraceptives has been found. Breast cancers diagnosed in women who have used oral contraceptives are more likely to be localized to the breast; therefore, overall, there is no significant difference in the cumulative risk of breast cancer between those who have used oral contraceptives and those who have never used them. There also is no evidence that OCP use offers protection. There is no evidence that oral contraceptive pills have any effect on colon cancer. Combined OCP use does not decrease the risk of cervical cancer; it increases it by about 2-fold, but only in long-term users (>5 years) with persistent human papilloma virus infection. Combined OCP use does not decrease the risk of hepatic cancer. Rarely, it has been associated with the development of benign or malignant hepatic tumors (adenoma and hepatocellular carcinoma). The risk increases with higher dosage, longer duration of use, and older age. Current estimates indicate there is about 2 times the risk of liver cancer after 4 - 8 years of use; therefore, protection by OCPs against ovarian cancer reduces the risk of gynecological cancer death.

A 28-year-old woman in her 2nd trimester of pregnancy presents with a 3.5 cm painless mass on her left breast. She has no other associated symptoms. She has a positive family history of breast cancer. She is afebrile; pulse is 80/min, and BP is 110/72 mmHg. On examination of the breasts, you notice engorgement and hypertrophy. The mass appears solid on ultrasonography (USG). Question What is the next step in evaluation of this mass? Answer Choices 1 Magnetic Resonance Imaging (MRI) 2 Percutaneous biopsy under local anesthesia 3 Fine needle aspiration cytology (FNAC) 4 Sentinel lymph node sampling using blue dye 5 Avoid any intervention at this time

Explanation The correct answer is percutaneous biopsy under local anesthesia. This is used to obtain a histological diagnosis, which can be done safely during pregnancy with a sensitivity of around 90%1. During pregnancy, non-ionizing examinations are preferred to those needing ionizing agents1. Breast ultrasonography is the first diagnostic instrument used by clinicians when a breast mass and the axillary area need to be assessed in a pregnant woman; it is non-ionizing and has high sensitivity and specificity1. MRI is incorrect; it should only be used when it will alter clinical decision making and when ultrasonography is inadequate1. No well-designed studies of the efficacy and safety of MRI of the breast during pregnancy have been reported, and results of some studies have shown that gadolinium-based MRI contrast agents pass through the placental barrier and enter fetal circulation1. Fine needle aspiration cytology (FNAC) should be done only in case of a cystic mass1. Gestational and puerperal hormones induce physiological hyperproliferative changes of the breast, which could lead to a false positive or false negative result with FNAC1. Therefore, this procedure is not recommended during pregnancy1. Staging with sentinel lymph node biopsy is done once the diagnosis of breast cancer is established. Clinicians can safely use sentinel lymph node staging during pregnancy using technetium 991. However, blue dye is associated with a risk of an anaphylactic maternal reaction, and it would probably distress the fetus; therefore, the use of blue dye should be avoided during pregnancy. Although sensitivity and specificity of sentinel lymph node biopsies during pregnancy have not been established, researchers have successfully used technetium-based identification in pregnant women1. Staging examinations and tumor biology assessment affect management decisions in pregnancy and should not be avoided.

An obese woman pregnant with twins is concerned about gaining too much weight. How do you counsel her regarding the Institute of Medicine's (IOM) recommended weight gain in women carrying twins? Answer Choices 1 A 25 to 42 pound weight gain is recommended. 2 A 37 to 54 pound weight gain is recommended. 3 An 11 to 20 pound weight gain is recommended. 4 She should maintain her current pre-pregnancy weight as much as possible. 5 A 25 to 35 pound weight gain is recommended.

Explanation The correct response is a 25 to 42 pound weight gain is recommended. The IOM did not distinguish different weight recommendations for women carrying twins from those for women pregnant with a singleton until recently. According to IOM, women pregnant with twins should attempt to gain between 37 - 54 pounds. A woman with a normal BMI (18.5 to 24.9) carrying only 1 baby should attempt to gain 25 to 35 pounds in pregnancy. IOM recommends that pregnant women with a low BMI (< 18.5) should attempt to gain 28 to 40 pounds if carrying 1 infant; overweight women with a high BMI (25 to 29.9) should attempt gaining 15 to 25 pounds, or 31 - 50 pounds if carrying twins; and obese women (BMI > 29.9) should gain 25 - 42 pounds if carrying twins, and only 11 - 20 pounds if pregnant with a singleton.

Your patient in the labor and delivery unit is a 26-year-old primigravida; she is at 36 weeks of gestation. She is experiencing fever and abdominal tenderness. Her pregnancy has been normal; however, 2 days ago she noticed clear fluid vaginal leaking. There are no changes in fetal movements, and there has been no vaginal bleeding. Today her temperature is 38.5 C, blood pressure is 120/80 mm Hg, heart rate is 92/min, and respirations are 18/min. Fetal heart rate is 160/min (tachycardia), with good variability. Contractions are registered every 10 minutes. On examination her abdomen is diffusely tender, the cervix is closed, and there is turbid vaginal pooling that shows an elevated pH and a ferning pattern. Question After administering antibiotics, what will be your next step in the management of this patient? Answer Choices 1 Cesarean section 2 Oxytocin 3 Magnesium sulfate 4 Indomethacin 5 Steroids

Explanation The correct response is oxytocin. Your patient most probably has chorioamnionitis secondary to the prolonged rupture of membranes. Delivery is indicated for the benefit of both mother and fetus. Her cervix is closed; therefore, the induction of labor is the best next step in the management. A cesarean section is not indicated in this case; instead, you should try vaginal delivery. You should only consider a cesarean section if there is significant maternal and/or fetal distress and a vaginal delivery is remote. Magnesium sulfate is not indicated; your patient has chorioamnionitis, and you do not want tocolysis. Indomethacin is not indicated; you do not want to delay the labor. Steroids (e.g., betamethasone) are used to improve fetal lung function and are given up to 34 weeks of gestation (the greatest benefit is at less than 32 weeks of gestation). Your patient is at 36 weeks of gestation; therefore, steroids are not indicated.

A 32-year-old woman in her 34th week of pregnancy comes to the labor floor after 2 - 3 hours of vaginal bleeding. Labor and vaginal delivery of her previous 3 pregnancies were normal. She is frightened, but oriented. While taking her medical history, what points are relevant to ask? Answer Choices 1 Cigarette use 2 The latest pap smear result 3 Recent sexual intercourse 4 Desire for future pregnancies 5 Recent vulvar pruritus

Explanation You should ask about recent sexual intercourse. The woman in this case exhibits 3rd-trimester bleeding, which is an obstetric emergency. History, physical exams, and lab test should be performed to assess maternal and fetal well-being. The differential diagnosis consists of placenta previa, abruptio placenta, and local vaginal or cervical bleeding from trauma. In medical history, you should ask about bleeding problems and sexual intercourse, as they can cause cervical trauma. In physical examinations, Leopold's Maneuvers outline the fetal lie and indicate fetal malposition. Postural hypotension suggests major uncompensated blood loss. After the history and physical exams have been completed, measurement of blood loss and urine output should be done. However, a type and cross match is an essential precaution. At this time, amniocentesis is appropriate to look for blood in the amniotic fluid and to determine fetal lung maturity. Internal examinations must not be performed in women suspected of having placenta previa, unless immediate delivery is required. The use of oxytocin challenge test in the face of 3rd-trimester bleeding is contraindicated. The best management for this woman would be careful monitoring of vaginal bleeding. Blood should be set up in the blood bank. Cesarean section may be required later, but not at this point. The use of Ritodrine to inhibit early labor is not indicated. Magnesium sulfate is used to control preeclampsia, however it is thought that magnesium sulfate can worsen bleeding in a woman with placenta previa by preventing blood coagulation.

Which one of the following bacteria is carried by about 20% of women of child-bearing age and is a cause of sepsis and meningitis of the newborn? Answer Choices 1 Lactobacilli species 2 Bacteroides 3 Bifidobacterium 4 Group B streptococci 5 Clostridium

Explanation Group B streptococci are carried by about 20% of women of childbearing age in their vagina and are a cause of sepsis and meningitis of the newborn and are acquired during passage through the birth canal. Lactobacilli, Bacteroides, Bifidobacterium and Clostridium are normally found in the colon and do not cause sepsis and meningitis of the newborn.

Examination of a newborn infant revealed the presence of hairy patch present on the lower back. Neural tube defect is most commonly associated with which of the following? Answer Choices 1 Teratogen exposure during second trimester 2 Radiation exposure during third trimester 3 Maternal infections during the third trimester 4 Increased maternal age at first pregnancy 5 Maternal folic acid deficiency in the first trimester

Explanation Neural tube defects are most commonly associated with maternal folic acid deficiency in the first trimester. Hence, 400mg of folic acid is recommended when attempting to get pregnant and during early pregnancy to prevent neural tube defects. Neural tube defects (NTDs) result due to the failure of closure of neural tube during the 4th week of embryogenesis. NTDs include anencephaly, spina bifida, and encephalocele and may also cause severe disabling birth defects. Congenital malformations of the spinal cord are the result of improper closure of the caudal neuropore. Malformations of the neural tube involving the spinal cord and vertebral arches are termed as spina bifida. The non-fusion of the spine can occur in varying degrees of severity ranging from a lack of bone fusion to protrusion of the meninges with or without nerve involvement. Severe type of spina bifida (meningomyelocele) involves protrusion of the spinal cord along with the meninges through a defect in the vertebral arch. The clinical manifestation vary depending on the level of the lesion; nerve involvement results in paralysis of the legs, urinary and fecal incontinence, skin anesthesia, and abnormalities of the pelvis, knees, and feet. Display of a hairy patch, a dimple, or hyperpigmentation may also be present. Early prenatal diagnosis by ultrasound and alpha-fetoprotein level estimation is essential for proper management. Though maternal folic acid deficiency plays a pivotal role in NTDs, the etiology of neural tube defect remains multifactorial. Maternal infections, radiation, and teratogen exposure predominantly in the early pregnancy are associated with NTDs. Increased maternal age is associated with Down syndrome.

A 20-year-old female (G1, P0) presents to your office for her 36th week antenatal care visit. The woman's blood group is O, Rh negative. Her husband's blood type is B, Rh positive. The fetal ultrasound showed mild enlargement in the cardiac chambers and pericardial effusion. The spectrophotometric analysis of the amniotic fluid sample showed increased absorbance at 450nm. The most likely mechanism regarding these findings is Answer Choices 1 Cardiac malformation 2 Hemolytic disease in the fetus 3 Hepatic dysfunction in the fetus 4 Hypoglycemia in the fetus 5 Polycystic kidney

Explanation The fact that the mother is Rh negative and that the father is Rh positive means that the likelihood that the mother will be carrying an Rh positive baby can either 0% or 50% depending on whether the father is heterozygous or homozygous for the Rh gene locus. The antenatal screening of the mother at 36 weeks gestation showed dilatation of the cardiac chambers and pericardial effusion in the fetus, which are signs of heart failure. Since the mother is Rh negative and has likelihood of 50% of carrying an Rh positive baby, hemolytic disease of the newborn should be on the top of your list while working up the differential diagnosis in this case. The presence of hyperbilirubinemia in the amniotic fluid as detected by spectrophotometry also should steer you towards hemolytic disease in the fetus due to Rh incompatibility with alloimmunization of the mother. It is a common misunderstanding that the first pregnancy should be safe in an RH neg. mother carrying and RH pos. fetus. Feto-maternal transfusion on a small scale can occur during pregnancy and depending on how much has been exchanged between the maternal and the fetal circulation, RH antibodies can be found in third trimester of primigravidas, that is why re-screening for RH antibodies around the 28th week is recommended. The heart failure and jaundice are not signs of hypoglycemia in the fetus and the mother is not diabetic making this possibility highly unlikely. The ultrasonic evaluation of the fetus showed only mild signs of heart failure and did not indicate the presence of cardiac malformation in the fetus. In addition the Rh-negative status of the mother should alert you to the possibility of an Rh incompatibility specially knowing that the husband is Rh positive. Hepatic dysfunction also will be associated with increased bilirubin in the amniotic fluid but will not explain the cardiac findings. The ultrasound examination would have detected the polycystic kidneys if it were present.

You are the resident on call in the ER when a multiparous patient presents in the first stage of labor, with contractions less than one minute apart. The chief resident calls you to assist in the delivery of the child and states that "the fetal head is engaged." In reference to this phrase, which of the following is true? Answer Choices 1 the widest portion of the fetal head has successfully passed through the pelvic inlet 2 forceps will need to be used because the widest portion of the fetal head is engaged at the pelvic inlet 3 in a multiple birth, the head of the first fetus is not presenting first, but is positioned correctly 4 the fetus is presenting in a frank breech fashion with the head positioned correctly in the pelvis 5 the crown of the head of the fetus is seen to bulge the perineum indicating birth is imminent

Explanation When the fetal head is engaged, the widest portion of the fetal head has successfully passed through the pelvic inlet. This phrase signals that the delivery of the infant is proceeding normally, that the presentation is head first, and is not a breech presentation. After this stage, with the complete dilation and effacement of the cervix, the child will descend down the birth canal. Only at that time would the fetal head be noted to be crowning, e.g., presenting as a bulge at the perineum.

You are providing care to a woman who is at 33 weeks gestation. Her pregnancy is complicated by gestational diabetes. She is being provided education by the dietician, and has weekly obstetrical appointments. What fasting blood sugar (FBS) readings should necessitate switching from diet control to insulin therapy? A FBS > 70 mg/dL B FBS > 90 mg/dL C FBS > 95 mg/dL D FBS > 110 mg/dL E FBS > 126 mg/dL

FBS >95 mg/dL Gestational diabetes has different risks associated with it, in particular for the fetus. Stricter glycemic controls are recommended for pregnant versus non pregnant women by the ACOG and the ADA.

An 18-year-old woman presents for advice regarding emergency contraception. She had sexual intercourse the day prior to presentation; she did not use contraception, and she wants to know if it is possible that she is pregnant. Her last menstrual period (LMP) began 5 days prior to presentation. She usually gets her menses once in 28 days and the period usually lasts for 5 days. The physical exam is normal. Question What hormone is responsible for the stimulation of the ovaries at this point of the menstrual period? Answer Choices 1 Estrogen 2 Follicle-stimulating hormone (FSH) 3 Inhibins 4 Luteinizing hormone (LH) 5 Progesterone

FSH Explanation The 5th day of the menstrual period corresponds to the follicular phase of the cycle, which starts from the 1st day of bleeding to the day of the LH peak. In this phase, the FSH, which is produced by the pituitary, stimulates the development of follicles in the ovaries; only 1 follicle is dominant at the end. During the 2nd part of this phase, the granulosa cells of the follicle begin to produce estrogen. Estrogen is produced by the granulosa cells of the follicle; they are very important in the follicular phase. Their level is very low at the beginning of this phase and reaches its maximal point at the middle of the cycle, just before the the LH peak. The cells are the feedback for the LH production, and in part, for the FSH production. Inhibins are also hormones produced by granulosa cells of the follicles; they are part of the negative feedback for the FSH. Inhibin B levels rise during the luteal-follicular transition, are highest during the mid follicular phase, go up again during the LH peak, and finally decrease in the late follicular phase. Inhibins A levels decrease during the late luteal phase. Ovulation occurs approximately at the middle of the menstrual cycle (day 14). At the end of the follicular phase a rise in the level of estrogens takes place, preceding the LH peak. Ovulation will occur an average of 30 hours after this peak. The 2nd 1/2 of the menstrual cycle corresponds to the luteal phase of the menstrual cycle which starts from the day of ovulation to the 1st day of menstruation. In this phase, the ovarian corpus luteum begins to produce progesterone. If there is fertilization of the ovum, the production of progesterone continues for 4 to 5 weeks until the placenta can produce a sufficient quantity of the hormone to maintain the pregnancy. If there is no fertilization, progesterone levels decrease and menstruation occurs.

A 20-year-old Lebanese woman presents to a family practice office because she wants to start birth control. She has never been sexually active, and she is engaged to be married in 2 months. She feels well and has no complaints. She thinks she wants "the pill". Her fiancé is also a virgin, and they are not interested in condoms or other barrier methods of contraception. She wants to delay childbearing for at least 2 years. A summary of her past medical history includes: Medications: occasional over-the-counter ibuprofen for menstrual cramps and headaches Allergies: Penicillin Surgical history: Tonsillectomy Medical history: No known conditions. OB/GYN history: Menarche age 12. Regular monthly menses, with mild-moderate dysmenorrhea. Family history: Patient's older sister had a blood clot in her lung that followed delivery of a child. Her paternal grandfather has diabetes and hypertension. Her maternal grandmother had a stroke. Her mother had a deep venous thrombosis in a leg, and her maternal grandfather had prostate cancer. Social history: The patient works as a waitress part-time and is attending college. She currently lives in a dorm. She denies the use of tobacco, alcohol, and recreational drugs. Vitals are obtained from your medical assistant prior to physical exam. Weight 123 lbs. Height 64" Body mass index (BMI) 21.1 Pulse 88 Blood pressure 134/86 Temperature 97.9 °F Question Based upon this patient's presentation, what test should be ordered before prescribing birth control pills? Answer Choices 1 Bleeding time 2 Complete blood count (CBC) 3 Complete metabolic panel (CMP) 4 Factor V Leiden 5 Pap smear

Factor V Leiden Explanation Generally, no screening tests are recommended prior to starting most patients on most forms of contraceptives. However, when risk factors suggest a possible contraindication, the clinician should delve deeper and individualize testing. In this case, the patient is healthy, but her family history is concerning for a heritable form of hypercoagulability. She should be tested for a Factor V Leiden mutation. Factor V Leiden mutations are estimated to be present in approximately 5% of the Caucasian population, and it is highest in those of Lebanese ethnicity, with a prevalence of over 14% of the affected population. This mutation is the most common of heritable causes of hypercoagulability. If the patient tests positive, then she should not use estrogens in either contraceptives or menopausal hormone therapy, due to increased risk of thromboembolism. This patient should be offered alternate methods of contraception. Bleeding time is a test of platelet function, not coagulation factors. Bleeding time is useful in evaluation of patients at risk of bleeding too much, not clotting. A complete blood count (CBC) is useful in evaluating several conditions, such as anemia and infection and alterations of platelet numbers. If the clinician ordered a CBC and it returned normal, it could provide false reassurance that this patient could safely use combination hormonal contraceptives; the CBC can be entirely normal in an individual with a Factor V Leiden mutation. There is another condition of inherited hypercoagulability, familial or essential thrombocytosis, which would show up on the CBC, but this condition is very rare; also, it often presents at birth, so this patient is unlikely to have it. A complete metabolic panel (CMP) provides information on liver and kidney function, glucose levels, and electrolytes. It does not provide information related to this patient's family history of hypercoagulability. Theoretically, the CMP could be done to ensure normal liver enzymes because many of the hormonal contraceptives are hepatically metabolized. However, routine testing of liver function is not recommended prior to starting birth control. The Pap smear (short for Papanicolaou) is a test for cervical cancer and pre-cancerous dysplasia. Many years ago, it was thought that birth control increased risk of cervical cancer. That myth has been disproved. There is no compelling reason to obtain a Pap smear in order to initiate birth control. Routine cervical cancer screening guidelines should be followed, which would indicate that this patient should wait until the age of 21 years before beginning Pap testing.

A 26-year-old female presents to the clinic for evaluation of her inability to conceive for 12 months. She menstruates monthly. Her past medical history is significant for PID x 2 and mild asthma. Her GC, Chlamydia, and pap smear are negative. Her TSH is 3.0 and her UCG today is negative. What is the most likely cause for her inability to conceive? A Anovulatory cycles B Hypothalamic state C Fallopian tube scarring D Hypothyroidism E Polycystic ovarian syndrome

Fallopian tube scarring C In a young and otherwise healthy female with a negative work up and a history of PID, the most common cause of infertility is fallopian tube scarring secondary to STD and PID.

A 29-year-old woman, G0P0, and her husband present to your office after 1 year of infertility. Their histories elicit irregular menses in the woman and that the man is an avid cyclist. Apart from semen analysis in the man, what laboratory and/or diagnostic studies should be pursued in the woman to further evaluate this couple's infertility? Answer Choices 1 Fasting prolactin, TSH 2 TSH, LH, T3, T4 3 Fasting prolactin, FSH 4 FSH, T3, T4 5 Fasting prolactin, TSH, FSH, LH, hysterosalpingogram

Fasting prolactin, TSH, FSH, LH, hysterosalpingogram Explanation Fasting prolactin, TSH, FSH, and LH should all be obtained in the female patient as her irregular menses more or less indicate a high probability of oligomenorrhea. It is also important to evaluate by hysterosalpingogram to look for evidence of uterine and/or tubal defects. It is also important to evaluate the male patient's semen to investigate male infertility related to oligospermia or hypomotility (1).

A 32-year-old, vegetarian female in mid-pregnancy complains of lack of energy and says she becomes easily fatigued. Upon any strenuous movement, her heart pounds rapidly and she becomes short of breath. Nutritional supplement of what mineral may alleviate the female's symptoms? Answer Choices 1 Mg2+ 2 Ca2+ 3 Fe2+ 4 Cu2+ 5 Zn2+

Fe2+ Explanation Pregnant females can experience iron deficiency anemia due to increased demands on their blood. Oxygen (O2) in the lungs binds to the iron ion, Fe2+ while complexed with the heme cofactor of hemoglobin in red blood cells. Iron supplements or foods with abundant iron, such as liver, lean meats, or vegetarian alternatives such as spinach, carrots, and raisins can alleviate anemic symptoms. The other minerals have biological roles but are not associated with anemic symptoms. Magnesium (Mg2+) coordinates with the negatively charged backbone of DNA and interacts with neurotransmitter receptors at excitatory synapses in the central nervous system. Mg2+ deficiency affects the nervous system, resulting in vasodilation, tremors, and depression. Calcium phosphate forms a hard material in bone and teeth. In addition, Ca2+ is a ubiquitous second messenger ion in cellular signaling coupled to G-protein signaling, hormone signaling, and ion channel activity. Ca2+ deficiency can give rise to muscle twitching or cramping and cardiac arrhythmias. Copper (Cu2+) participates in bone and blood formation and is an electron carrier in mitochondrial electron transfer proteins. Cu2+ deficiency is uncommon since the trace amounts needed are satisfied by most diets. Zinc (Zn2+) is a cofactor of many DNA and RNA binding proteins, including many transcription factors. Severe zinc deficiency can retard growth in children, can cause low sperm count in males, and can slow wound healing.

A nulliparous 25-year-old woman has had an uneventful, spontaneous labor at term. She has an epidural anesthetic and has been in the second stage of labor for 2 hours. The fetal heart tones show some signs of deterioration. You decide to expedite the delivery with the use of forceps. What criterion defines the application of low forceps? Answer Choices 1 Fetal head is on the perineum 2 Fetal skull has reached the pelvic floor 3 Fetal skull is at station +2, but not on the pelvic floor 4 Head is engaged, but above station +2 5 Scalp is visible at the introitus without manually separating the labia

Fetal skull is at station +2, but not on the pelvic floor Explanation Low forceps is defined by the skull being at station +2 or greater, but not on the pelvic floor. Outlet forceps includes situations where the scalp is visible, the fetal skull has reached the pelvic floor, or the head is on the perineum. In addition, rotation must not exceed 45 degrees. Midforceps is the head engaged but above a station of +2.

Your patient is a full-term newborn with facial defects affecting her eyes, nose, and upper lips. She is the first child of non-consanguineous parents. Her mother has a history of gestational diabetes, which began at the start of her pregnancy. She contracted German measles a month before the delivery. During the course of infection, her self-prescribed daily treatment was 3 tablets of aspirin, at least 6 cups of herbal tea, and a double dose of folic acid. Imaging studies showed prosencephaly. Question What risk factor is responsible for the condition of this child? Answer Choices 1 Rubella 2 Aspirin 3 Herbal tea 4 Gestational diabetes 5 Folic acid

Gestational diabetes Explanation The only possible risk factors during the period of the development of face and forebrain is gestational diabetes, because the forebrain is formed and the face begins to develop in the fifth and sixth weeks of human pregnancy. Other known risk factors for prosencephaly include transplacental infections, bleeding during the first trimester, a history of miscarriages, use of some drugs potentially unsafe in pregnancy (insulin, birth control pills, lithium, anticonvulsants, retinoic acid, cholesterol-lowering agents, and maternal hypocholesterolemia), and toxins (alcohol, nicotine). Rubella, or German measles, is not likely the cause of birth defects because infection occurred after organogenesis was finished. Rubella can cause miscarriage, stillbirth, or birth defects (most often deafness, brain damage, heart defects, and cataracts) if infection occurs during the first 16 weeks of pregnancy. Aspirin taken during the last trimester may increase the risk of bleeding in the newborn, but cannot cause birth defects. Herbal tea is not the cause of birth defects after organogenesis is finished. All women capable of pregnancy should take a daily vitamin supplement of folic acid to prevent neural tube defects. In the late pregnancy, however, folic acid can neither prevent nor cause birth defects of the brain and face.

Some temporary disease processes are unique to pregnancy and may occur in one or all of a women's pregnancies. Which of the following would raise suspicion for the first phase of toxemia referred to as pre-eclampsia? Answer Choices 1 Headache, visual disturbances, abdominal pain, vomiting 2 Sinus pain, chest pain, bowel disturbances 3 Chest pain, shortness of breath, pain in arms 4 Bowel disturbances, pain in arms, pain in back 5 Face swelling, edema of the limbs, renal failure

Headache, visual disturbances, abdominal pain, vomiting Explanation Pre-eclampsia, the first stage of toxemia, exhibits the symptoms of headache, visual disturbances, abdominal pain, and vomiting. These are often overlooked as normal pregnancy symptoms. The patient should be monitored for further symptoms. Chest pain, shortness of breath, and pain in the arms would raise the suspicion of cardiac problems and should not be ignored.

A 37-year-old female presents to the clinic for her obstetrical appointment. She is a G5P4 African American female. On physical exam her uterus is larger than expected for dates by 5 cm. What is her risk of having twins increased by? A Her increasing age B Her increasing parity C Her race D Her use of clomifine E Her late age of menarche

Her use of clomifine Incidence of multiples is not related to age, parity, or menarche; the increase seen in the last few decades is solely related to the increased use of fertility drugs.

A 27-year-old woman presents with cramping abdominal pain and vaginal bleeding. Further history reveals amenorrhea for the past 2 menstrual cycles. On examination, she is found to have left lower abdominal tenderness and an adnexal mass. Lab values reveal an elevated Beta HCG level. Question A further history would possibly reveal what associated risk factor? Answer Choices 1 History of PID 2 Diaphragm use 3 Use of condoms 4 Oral contraceptive pills 5 Multiple sexual partners

History of PID Explanation A history of pelvic inflammatory disease is the correct answer. The vignette describes a potential ectopic pregnancy, evidenced by the triad of amenorrhea, adnexal mass, and an elevated B-HCG. Inflammation of the fallopian tubes secondary to bacterial infection resolves with fibrosis and adhesion formation. The fertilized egg gets trapped on its transit through the fallopian tube, resulting in an ectopic pregnancy. Being thin walled, implantation in the fallopian tube invariably results in a rupture. Diaphragm use is incorrect. Diaphragms sit atop the cervix, preventing sperm entering the uterine cavity. They are used as a temporary measure and are not associated with ectopic implantation. Condom use is incorrect. It is used as a temporary measure, and condom use is not associated with ectopic implantation. Oral contraceptive use is incorrect. Regular OCP use is associated with an extremely low failure rate. In the event of contraceptive failure, there is no increased risk of ectopy with OCP use. Multiple sexual partners is incorrect. While a common risk factor for PID, by itself it is not a risk factor.

A 24-year-old gravid 3 para 0 ab 2 presents complaining of vaginal pressure every 2 hours, but no pain. She is at 26 weeks gestation. This pregnancy is uncomplicated to date. On physical exam she is in no acute distress and her vitals are normal. She is placed on a fetal monitor and no contractions are noted; fetal heart tones are 138. A sterile speculum exam reveals her cervix to be dilated to 4 cm. What is the most likely diagnosis? A Preterm labor B Group B Streptococcus C Incompetent cervix D Braxton Hicks contraction E Chlamydia trachomatis

Incompetent cervix While infection is a significant cause of premature labor, it is not a cause of incompetent cervix. Incompetent cervix is most likely found in the presence of recurrent pregnancy loss and painless dilatation.

A 27-year-old woman has a history of recurrent ovarian cysts; she is being treated with a combination oral contraceptive (norethindrone/ethinyl estradiol 1mg/35mcg). The norethindrone in this drug acts to suppress ovulation by what process? Answer Choices 1 Decreasing circulating sex-hormone binding globulin (SHBG) 2 Increasing gonadotropin production 3 Inhibiting release of follicle-stimulating hormone (FSH) secretion from the anterior pituitary 4 Inhibiting release of luteinizing hormone (LH) secretion from the anterior pituitary 5 Stimulating proliferation in the endometrial lining

Inhibiting release of luteinizing hormone (LH) secretion from the anterior pituitary Explanation Norethindrone is the progestin component of the oral contraceptive, which "primarily suppresses luteinizing hormone (LH) secretion (and thus prevents ovulation)."1 The LH 'surge' is responsible for triggering ovulation. The LH is released from the anterior pituitary and is suppressed in a negative feedback cycle when progestin levels are increased. The norethindrone would be expected to increase circulating sex-hormone binding globulin (SHBG), resulting in decreased free androgens in the serum.2 Both the estrogen and progestin components of this drug decrease gonadotropin (LH and FSH) production by negative feedback to the anterior pituitary. The estrogenic agents in oral contraceptives (ethinyl estradiol) suppress follicle-stimulating hormone (FSH), as well as provide stability to the endometrium and potentiation of the progestin's actions. The effect on FSH is primarily attributed to the estrogen, while the effect on LH is primarily attributed to the progestin, such as norethindrone. Estrogen stimulates proliferation in the endometrial lining. Progestins, such as norethindrone, are associated with endometrial thinning and a shift to the secretory phase.1Neither effect on the endometrium leads to ovulation suppression.

A 27-year-old woman and her male partner come to the emergency department for assistance with emergency contraception. They experienced condom failure during intercourse an hour ago and neither desires pregnancy. Her last menstrual period was approximately two weeks ago and her cycles occur every 28-30 days. Her medical history includes a deep venous thrombosis during labor and delivery 5 years ago. What is the most appropriate course of action at this time? A Administration of an ethinyl estradiol and levonorgestrel combination now and in 12 hours B Dilation and curettage C Insertion of a copper-containing intrauterine device D Serial beta hCG determinations E Testing for factor V Leiden

Insertion of a copper-containing intrauterine device Insertion of a copper-containing IUD is an effective means of preventing an unintended pregnancy in this case. The woman's history of clotting is a contraindication to use of combination oral contraceptives (A). Dilation and curettage (B) is neither appropriate nor effective for emergency contraception. Performing beta hCG determinations (D) would merely detect pregnancy if it were to occur. Given her history of clotting, testing for factor V Leiden (E) may be appropriate but will not affect management at this time.

What hormone is responsible for ovulation? Answer Choices 1 Follicle stimulating hormone 2 Luteinizing hormone 3 Estrogen 4 Testosterone 5 Insulin-like growth factor

LH Explanation Luteinizing hormone is the hormone responsible for initiation of ovulation. Follicle stimulating hormone is responsible for follicle maturation. Estrogen increases the number of luteinizing hormone (LH) receptors, which also also leads to unregulation of LH. Ultimately, a rapid rise in LH (LH surge) occurs approximately 24 - 36 hours prior to ovulation. Estrogenis produced by the conversion of testosterone to estradiol by aromatase in the granulosa cells.

A 30-year-old woman comes in for evaluation of infertility. She and her husband have been having unprotected intercourse for the past year. Her menstrual cramps have become increasingly painful, and she has a severe low backache for several days before and during her menses. She complains that intercourse is painful when her husband "goes deep." Physical examination reveals multiple tender nodules of various sizes in the posterior vaginal fornix. Definitive diagnosis is best accomplished using what methodology? A Abdominal radiography B CA-125 measurement C CT scanning of the abdomen D Laparoscopy E Pelvic ultrasonography

Laparoscopy D This woman has a classic presentation for endometriosis. Definitive or final diagnosis can only be made at laparoscopy or laparotomy, allowing direct visualization of the endometrial implants. Abdominal radiography (A), computed tomography scanning (C), and ultrasonography (E) are not usually helpful in making a definitive diagnosis. CA-125 (B) may be elevated in endometriosis, but lacks specificity.

A 33-year-old gravida 3, para 1011, whose last menstrual period (LMP) was 6 weeks ago, presents with a 3-day history of vaginal bleeding as well as intermittent, non-radiating right lower quadrant pain. She states that her pain scale is a 7/10 and that the pain is worse today than it has been previously. Her past medical history is significant for gonorrhea at age 16; she was treated with antibiotics. Vital signs are: BP, 80/42 mm Hg; pulse is 120 BPM, and respirations are 22/min. The patient is afebrile, and oxygen saturation is 95% on room air. The abdomen is distended, and bowel sounds are absent. There is rebound tenderness and guarding. Pelvic exam demonstrates a mass in the right adnexa. The uterus is approximately 4 - 6 weeks size. A serum beta human chorionic gonadotrophin (HCG) level is 3,723 mIU/ml. Hematocrit is 24%. Ultrasound demonstrates free fluid in the pelvis, and there is a 6 cm complex mass in the right adnexa. There is no evidence of an intrauterine gestation. Question What is the most appropriate treatment for the patient at this time? Answer Choices 1 Laparoscopic salpingostomy 2 Methotrexate, 75 mg IM x 1 dose 3 Serial quantitative beta HCG levels every 48 hours 4 Laparotomy 5 Laparoscopic right salpingo-oophorectomy

Laparotomy Explanation Approximately 2% of all pregnancies are ectopic pregnancies, and most are located in the Fallopian tube. The triad of amenorrhea, vaginal spotting, and unilateral adnexal pain is strongly suggestive of an ectopic pregnancy. This patient is hemodynamically unstable. Such patients should undergo exploratory laparotomy as soon as is possible after the presumptive diagnosis has been made. Laparoscopic surgery of any kind is unacceptable in the face of a hemodynamically unstable patient with an ectopic pregnancy. Methotrexate is an acceptable alternative to surgical management of an ectopic pregnancy, but is contraindicated when an ectopic pregnancy is ruptured, or when the adnexal mass is >3.5 cm on ultrasound. Serial quantitative beta HCG levels may be obtained every 48 hours in order to confirm the diagnosis of ectopic pregnancy; however, in this case, the patient's unstable status makes this treatment plan dangerous and unnecessary.

A 34-year-old diabetic female complains of amenorrhea for the past 2 months. Last week she tested positive for strep pharyngitis and is currently being treated. She admits that she has not been using birth control. Her urine HCG is positive. Her current medications are listed in the choices below. Which of the following medications should you discontinue? A amoxicillin B lisinopril C acetaminophen D humalog insulin E methyldopa

Lisinopril B The correct answer is (B). Lisinopril, an ACE inhibitor, is contraindicated in pregnancy due to known problems with fetal toxicity and should be stopped as soon as possible once pregnancy is confirmed. If a patient is planning on becoming pregnant the ACE inhibitor should also be discontinued. ARBs should also be avoided. Choices (A), (C), and (D), and (E) can be used safely in pregnancy and are considered category B. Methyldopa is preferred in the treatment of hypertension in pregnancy and its safety is supported by evidence.

A 24-year-old female presents with hyperpigmented macules on her cheeks, nose, and upper lip. They have been present for a couple of months. Her current medications include oral LoEstrin 24 Fe, cetirizine, and a multivitamin daily. What is the most likely diagnosis? A congenital nevus B melasma C post-inflammatory hyperpigmentation D café-au-lait macule

Melasma The patient is experiencing melasma secondary to the use of oral contraceptives. This is a frequent cause of melasma. Melasma can also be precipitated by hormonal changes that occur during pregnancy. The condition will resolve upon discontinuation of the oral contraceptive. A congenital nevus is a nevus that presents within the first year of life. It is monitored in the same way as acquired nevi. They can be larger than acquired nevi, with only a slight increase in chance of malignant change over time. Post-inflammatory hyperpigmentation includes darker areas of pigmentation that can result after inflammation on the skin. Common causes include acne and atopic dermatitis. The hyperpigmentation will resolve over time. A Café-au-lait macule is a type of birthmark. It is usually light tan to light brown in appearance, and can vary greatly in size. They are usually benign, but can be associated with neurofibromatosis when more than six, with a diameter greater than 1.5 cm, are present.

A 27-year-old woman complains of years of menstrual irregularity and increasing facial and chest hair. PMH: significant for ovarian cyst and left cytectomy. She is a non-smoker and non-drinker. Labs include a negative uCG, elevated LH, and low FSH. She desires fertility and she has not responded to three cycles of clomiphene. What would be the next choice of medication that may return ovulation? A Insulin B Metformin C Dexamethasone D Spironalactone E Finasteride

Metformin B Dexamethasone, finasteride, and spironalactone all treat symptoms of hirsutism, but do not treat the underlying cause of PCO or improve fertility outcomes. PCO has an underlying insulin resistance that can be treated with oral hypoglycemics and improve sensitivity to insulin. Adding insulin does not improve the resistance.

A 24-year-old G2P2 delivered a viable female infant (8 lb 4 oz) via caesarean section, after a failed 20-hour induction for post date pregnancy. On day 2, she developed a postoperative fever of 101F (38.3C). She had slightly increasing abdominal cramping and pain, no change in loci, is voiding well, and has passed flatulence. Her WBC is 19,000. What is the mostly likely cause for her fever? A Urinary tract infection B Ileus C Metritis D Atelectasis E Tubo-ovarian abscess

Metritis C The patient is passing urine and flatulence well, making choices A and B less likely. Fever greater the 38C is the most important indicator of metritis. Fever is not usually indicative of mild atelectasis. Tubo-ovarian abscess is usually a complication from PID. The patient has many risk factors for metritis including c-sect, prolonged induction, and fever.

While performing a Doppler ultrasound on a woman at 30 weeks gestation, you notice that the fetal heart rate is consistently within the range of 130 - 140 beats/minute. What can you conclude about the heart rate of the fetus? Answer Choices 1 Structural heart disease 2 Normal heart rate 3 Asphyxia 4 Bradycardia 5 Tachycardia

Normal HR Explanation The fetus has a normal fetal heart rate, as it is within the range of 110 - 160 beats/minute. The diagnosis of structural heart disease cannot be established by a fetal heart rate. The fetus has neither tachycardia (an early sign of fetal asphyxia), nor bradycardia (a late sign of asphyxia). Fetal bradycardia is defined as a decrease in the baseline fetal heart rate to <100 beats per minute. Suspicious tachycardia is defined as the heart rate being between 150 and 170; a pathological pattern is above 170 beats/minute.

A 23-year-old G1P0 woman presents for her first prenatal visit. The first day of her last menstrual period (LMP) was February 23. Generally, she feels well and has no complaints. The patient has started a prenatal vitamin, is getting the appropriate amount of sleep, and is keeping up with a moderate exercise routine. A complete physical exam is performed, and there are no abnormal findings. Initial ultrasound documents fetal cardiac activity. Question Using Nägele's rule, what is the patient's estimated date of confinement (EDC)? Answer Choices 1 November 16 2 November 30 3 November 23 4 September 23 5 September 30

November 30 Explanation Nägele's rule is a mathematical way of calculating a pregnant patient's EDC using the first date of her LMP: EDC = LMP - 3 months + 7 days. Therefore, using this equation, this patient's EDC is November 30.

A woman who is pregnant suffered a spontaneous abortion at 12 weeks gestation. She is now a G2P1Ab1 and is Rh negative. When should she receive her next Rhogam (Rho D immune globulin) shot? A Now B In one month C At conception of her next pregnancy D 28 weeks gestation of next pregnancy E After delivery of her next viable infant

Now A Placental implantation occurred and separated with the spontaneous miscarriage. Therefore, there is a slight chance of isoimmunization, so Rhogam should be given now so that the mother does not develop antigens that can cross the placenta during the first half of the next pregnancy.

A 33-year-old G1P0 presents for evaluation of her inability to conceive a pregnancy for six months. She menstruates monthly. Her past medical history is significant for PID x 2, for which she was hospitalized for IV antibiotics. Her Chlamydia, GC, and pap smear are normal. When should a more comprehensive evaluation for her infertility begin? A Now B 6 months C 12 months D 18 months E 24 months

Now A comprehensive work up should begin now, due to her advancing age and history of significant PID, which may require surgical treatment.

A 51-year-old female presents to her primary care provider for her annual physical. She is a healthy white female and a non-smoker. She has mild HTN, but an otherwise negative health Hx. In addition, FHx is negative and ROS is negative. Her LMP was 6 months ago. Her last mammogram was 3 years ago. When should she have her next mammogram? A In 2 years B In 1 year C Now D Only if self breast exam reveals abnormality E Only if provider breast exam reveals abnormality

Now C The preponderance of data strongly supports the benefits of a screening mammography. New analyses of older randomized studies have suggested that screening may not work. While the design defects in some older studies cannot be retrospectively corrected, most experts, including panels of the American Society of Clinical Oncology and the American Cancer Society, continue to believe that screening conveys substantial benefit.

A patient seen at the prenatal clinic develops Graves disease at 25 weeks' gestation. Which of the following is the most appropriate treatment? A PTU 100 mg po tid B methimazole 10 to 30 mg po qd C propranolol 80 mg po qid D radioactive iodine therapy (RAI, 131I) E levothyroxine 0.1 mg po qd

PTU 100 mg po tid In nonpregnant patients, PTU and methimazole are the drugs of choice for the management of Graves disease. During pregnancy, PTU has a lower incidence of crossing the placental barrier than does methimazole. It also is excreted into breast milk to a lesser degree than is methimazole. Propranolol will help with the symptoms of Graves but not treat it. It can also cause low birth weight in the infant. RAI is contraindicated in pregnancy. Levothyroxine will worsen a Graves patient's hyperthyroidism.

A 37-year-old G3P2 female at 39 weeks gestation presents to the labor and delivery unit complaining of abdominal pain. Laboratory evaluation reveals anemia coagulopathy that is felt to be consumptive. What is the most likely diagnosis to have caused this? A Placental abruption B Placenta previa C Preeclampsia D Labor E Pre-existing anemia

Placental abruption Reproductive placental abruption is the most common cause of coagulopathy in pregnancy; the hemorrhage may be concealed and is not always evident. In the presence of pain, anemia, and coagulopathy, abruption should be assumed until proved otherwise.

A 23-year-old primigravida is referred to her obstetrician by a community nurse. The nurse noted 2 blood pressure readings of 150/90 and 154/90 taken 15 minutes apart. Physical examination reveals a uterus at roughly 24 weeks gestation and grade 2 pitting edema. Urine analysis is 1+ positive for albumin. A sonogram at eight weeks gestation showed a single live intra-uterine gestation. She has had regular antenatal check-ups, and has no past history of hypertension or diabetes. Both her parents and grandparents are hypertensive. She is unaware of whether her mother or grandmother faced similar problems during their pregnancies. Question What is the most likely initiating event for her condition? Answer Choices 1 Over activation of her renin-angiotensin system 2 Overproduction of B-HCG 3 Placental ischemia 4 Renal artery stenosis 5 Essential hypertension

Placental ischemia Placental ischemia is the correct answer. The vignette describes a woman with classic features of pregnancy induced hypertension, as evidenced by the blood pressure readings, edema, time of onset beyond 20 weeks and proteinuria. The initiating event appears to be abnormal cytotrophooblast invasion of the spiral arterioles, which leads to activation of the maternal vascular endothelium. The cascade that follows includes enhanced formation of endothelin and thromboxane, increased vascular sensitivity to angiotensin II, reduced nitric oxide, and prostacyclin (vasodilator) synthesis. Over activation of her renin-angiotensin system is incorrect. While it is an important component of the response, it does not initiate the process. Overproduction of B-HCG is incorrect. It may result in high blood pressure as seen in hydatiform mole and multiple gestations. However, there is no indication of either in this vignette. Renal artery stenosis is incorrect. It is a common cause of hypertension in young individuals. It would be present through the duration of pregnancy. Essential hypertension is incorrect. Essential hypertension is a pathology associated with aging, the pathogenesis of which is unclear. It is certainly unlikely in a 23-year-old. In addition, the patient would have a past history of hypertension.

A 23-year-old G1P0 presents to the office complaining of headache, nausea, swelling, and generally not feeling well. She is at 33 weeks gestation. A physical exam reveals a 5-pound weight gain in 2 weeks, BP 148/90, P 84, T 98.1°F, and UA concentrated with 1+ protein. What is the most likely diagnosis? A Pregnancy induced hypertension B Preeclampsia C Eclampsia D Gastroenteritis E Third space fluid retention

Preeclampsia B Preeclampsia has a classic triad of weight gain, elevated blood pressure, and proteinuria. A woman with this triad of symptoms has preeclampsia until proven otherwise.

A 17-year-old female presents to the emergency department complaining of watery vaginal discharge for 6 hours. She is found to be at 35 weeks gestation. An external fetal monitor reveals fetal heart tones in the 130s, good variability, and no contractions. What is the most likely diagnosis? A Preterm labor B Preterm rupture of membranes C Premature rupture of membranes D Preterm labor and premature rupture of membranes E Preterm and premature rupture of membranes

Preterm and premature rupture of membranes Preterm rupture of membranes is defined as rupture before 37 weeks gestation; premature is defined as before the onset of labor. The absence of pain or contractions decreases the likelihood of labor.

A 25-year-old woman presents to discuss her options of available contraception. You review her history and note menses onset at 12 years old, duration of menses typically around 6 days in length, and it occurs every 30 days. She is G0P0, and she has no history of abnormal pap smears or diagnosed STIs. The patient is a non-smoker, single, and in a monogamous relationship with 1 partner for the past year. All other medical history is noncontributory with the exception of the patient having a deep venous thrombosis at age 19 and a pulmonary embolism at age 21. Other than anticoagulation therapy for the appropriate amount of time, no other hematological evaluation was pursued after these events. Question Given the most likely inherited diagnosis, what would be the safest form of contraception at this time? Answer Choices 1 Estrogen/progestin combination oral contraception pill 2 Progestin-only oral contraception pill 3 Etonogestrel/ethinyl estradiol vaginal ring 4 Norelgestromin/ethinyl estradiol patch-transdermal 5 Estradiol/medroxyprogesterone monthly injection

Progestin-only oral contraception pill Explanation The correct response is progestin-only oral contraception pill. Our patient above is overall healthy 25-year-old female patient. However, her history of multiple hypercoagulable events without any substantial risk factors puts her in the likely category of possessing an inherited hypercoagulable state. The most common hypercoagulable state that is inherited currently is Factor V Leiden. Despite the fact that this patient is very highly likely to have Factor V Leiden, she can still be offered a type of contraceptive method to prevent unplanned pregnancy. The common pre-thrombotic contraceptive component is estrogen. For this reason, the patient above would be safest at beginning any method of contraception that only contains progestin. There are several types of contraception that are progestin-only: progestin-only implants, a monthly injection of only progestin, levonorgestrel based intrauterine device, and the progestin-only pills, which are also referred to as the "mini-pill." The estrogen/progestin combination oral contraception pill, the etonogestrel/ethinyl estradiol vaginal ring, the norelgestromin/ethinyl estradiol patch-transdermal, and the estradiol/medroxyprogesterone monthly injection all contain a form of estrogen, which use of this should be strictly avoided in patients such as the one described in the above scenario.

A 36-year-old woman presents for her annual gynecological examination. In addition to her routine testing, she is seeking advice on contraception. She currently takes no medication and has no known medical conditions. She smokes a 1/2 pack of cigarettes daily; she drinks 1 - 3 alcoholic drinks weekly, and she is currently sexually active with 2 partners. Her past PAP smears have all been normal. She is G2P1 and is uncertain whether she wants children in the future. Question What is your recommendation for contraception at this time? Answer Choices 1 Combination estrogen-progestin pill 2 Progestin-only pill 3 Ortho Evra (ethinyl estradiol/norelgestromin) patch 4 NuvaRing (etonogestrel/ethinyl estradiol) 5 Tubal ligation

Progestin-only pill Explanation Out of the above options, the patient should receive progestin-only pills. The patient is a smoker, which is a contraindication for estrogen containing products. The patient should be informed that she needs to take progestin-only pills at the same time daily; otherwise, she needs to use a back-up form of contraception. The patient cannot receive the combination estrogen-progestin pill, the Ortho Evra (ethinyl estradiol/norelgestromin) patch, or the NuvaRing (etonogestrel/ethinyl estradiol); they all contain estrogen. The provider should not recommend tubal ligation because the patient is uncertain whether she wants children in the future; tubal ligation is considered a permanent sterilization option.

Your patient is a 26-year-old mother of two young children whose second child was just delivered two weeks ago and she is breast-feeding. She complains of insomnia and depression. She denies the use of any drugs or alcohol. You would like to hold off on pharmacologic therapy unless her symptoms progress. Which of the following will be your first line of treatment? A Encourage more time with the baby B Encourage her to begin planning to get back to work C Promote adequate sleep D Suggest she get out of the house more often E Suggest her husband help more around the house

Promote adequate sleep The correct answer is to promote adequate sleep (C). Postpartum "blues" are very common. Depression can occur in some cases, but most resolve without therapy. When required, SSRIs may be used even when a woman is breast feeding, though no studies have been done.

A 28-year-old G2P2 woman has just delivered a term male infant via normal spontaneous vaginal delivery. There was spontaneous rupture of the membranes 2 hours prior to delivery. Meconium stained amniotic fluid and thick particulate meconium was noted. The infant is brought under the radiant warmer where he appears peripherally cyanotic. He is crying and moving vigorously, and his heart rate is 160 beats per minute. Question What is the most appropriate next step in the resuscitation of this infant? Answer Choices 1 Bulb suction of the oropharynx and nasopharynx 2 Positive pressure ventilation via bag-valve mask 3 Removal of meconium by bulb suction 4 Routine neonatal evaluation since infant is vigorous 5 Suction the trachea via an endotracheal tube

Routine neonatal evaluation since infant is vigorous Explanation The recommendations have changed with the publication of the 2005 AHA/AAP Neonatal Resuscitation Program guidelines. Although the presence of meconium in the amniotic fluid should alert the caregiver to the possibility of neonatal distress, a vigorous infant is a good sign and further intervention is not warranted. Routine suctioning of the nose and mouth as the head is delivered is no longer recommended because it does not reduce the risk of meconium aspiration syndrome. In the presence of meconium stained fluid, suctioning is recommended if the infant has a low heart rate (<100 bpm), depressed respirations, or poor muscle tone. In an infant with one or more of these signs of distress, suctioning should be performed. Prior to drying, the infant should have visible, residual meconium removed from the nose and mouth by bulb suction. Then, under direct visualization, the trachea should be suctioned with an appropriately sized endotracheal tube. The infant in this case scenario is clearly vigorous and therefore does not require suctioning. He should only receive routine care (standard positioning, warming, etc.).

A 28-year-old pregnant woman who is currently at 33 weeks gestation is being evaluated for contractions at labor and delivery in the hospital. Laboratory testing on vaginal secretions from this patient reveals the following: Fetal fibronectin Positive Dried secretions on glass slide Ferning present Nitrazine paper Color change of yellow to blue Question What do these tests indicate? Answer Choices 1 Severe chorioamnionitis is present. 2 Fetal lungs are mature. 3 Rupture of membranes has taken place. 4 Estimated date of delivery (EDD) is confirmed. 5 The patient has a mild preeclampsia.

Rupture of membranes has taken place Explanation While direct observation of fluid leaking from the cervix is the best support, the above 3 tests (positive fetal fibronectin, ferning present on microscope slide and increased pH on nitrazine paper) indicate presence of amniotic fluid and thus rupture of membranes. Fetal fibronectin is not present in normal vaginal secretions. Normal vaginal pH during pregnancy is 4.5 - 4.7.1 However, amniotic fluid is 7 - 7.5, so after rupture of membranes, an alkaline pH is noted that causes corresponding color changes in the nitrazine paper. The laboratory testing that would most aid in a diagnosis of severe chorioamnionitis includes Gram stain for organisms and a white blood cell count of the amniotic fluid. 1 Often, the diagnosis is confirmed after pathology examination of the placenta. Fetal lung maturity is best evaluated with a lecithin to sphingomyelin (L:S) ratio. Some laboratories use different values for a level corresponding to fetal lung maturity (>/= to 2.0 or >3.0). A very low L:S ratio indicates fetal lungs are immature. 1 Confirmation of estimated date of delivery (EDD) cannot be accomplished with laboratory testing. Very early in a pregnancy, human chorionic gonadotropin (hCG) levels rise in a predictive manner in normal singleton, intrauterine pregnancy. 1 However, the level of hCG can neither be used to predict nor confirm the EDD. The best methods of confirming an accurate EDD include calculation based upon known date of last menstrual period (in a woman with regular menstrual cycles) and ultrasound in the 1st trimester.2 Preeclampsia is a syndrome in pregnancy consisting of hypertension and proteinuria. The testing that indicates a diagnosis of preeclampsia includes a blood pressure measurement and evaluation of the urine for protein, preferably from a 24-hour urine collection.3 None of the above listed tests on vaginal secretions indicate preeclampsia; furthermore, they do not indicate severity level.

A sexually active, 17-year-old patient presents alone to your office and requests contraceptive counseling. Because she is under the age of consent, what is an ethical concern of which you must be aware in seeing her? Answer Choices 1 You are obligated to refuse to see her. 2 You may only see her with parental consent. 3 You do not need parental consent if she is accompanied by an adult. 4 She can give consent for contraceptive counseling and evaluation of STDs. 5 You must counsel abstinence as contraception at her age.

She can give consent for contraceptive counseling and evaluation of STDs. Explanation This case highlights several ethical issues. Autonomy indicates that an individual may make his/her own health care decisions based on their own values. It is usually limited by a determination of whether the individual is competent and, in pediatrics, at least in part determined by the age of the individual. Competence is an individual's ability to understand the possible consequences of his/her decision and the available alternatives. Patients are generally considered to have a right to confidentiality, i.e., trust that information about them will not be disclosed. Depending on the age of the patient, there may be a conflict of interest between the physician's duty to the patient and the physician's duty to the parents. States either have laws allowing individuals who would otherwise be considered minors to obtain contraceptive services or have no law precluding the provision of such services.However, most states have placed the interests of the adolescents above those of the parents in issues relating to the management of sexually transmitted diseases and other issues of sexuality. Physicians have a duty to tell the truth to their patients. This duty can create quite a conflict when other family members, especially parents, request information about another family member that the physician is obligated to keep confidential.

A 40-year-old white male patient presents to your office to ask for advice regarding the inability of his wife to conceive. She is younger than he is by eight years and is believed by him to be in good health. Additionally, she has two children from a previous marriage, and both pregnancies were without complications. He was also married once before but his first wife did not conceive children. He has not yet undergone any fertility tests. The most important factor concerning male fertility is Answer Choices 1 Lung and liver dysfunctions 2 Spermatogenesis, varicocele, infections, drugs, and heat 3 Mucous and sperm interactions 4 Endometrial cavity and shape 5 The size of the genital tract 6 Personal and social history as it regards smoking, drugs and eating habits 7 Past infections of the male genitalia

Spermatogenesis, varicocele, infections, drugs, and heat Explanation Factors involved in fertility: The male factor---Spermatogenesis, infections (prostatitis, epididymis, mumps) varicocele, heat (decrease sperm count and motility) drugs (marijuana, tobacco, alcohol). The ovarian factor---Ovulation The cervical factors---Mucus and sperm interaction The uterine factor---Endometrial integrity and cavity size and shape The tubal factor---oviductal potency and anatomic relationships to the ovary The coital factor---Insemination

A 42-year-old gravida 1, para 1 presents with a 4-month history of menorrhagia. She is having shortened menstrual cycles that are sometimes only 15 days in length, with menstrual bleeding for 5 - 6 days. She is using approximately 12 - 14 pads or tampons per day. She admits to fatigue, headaches, and occasional dizziness, but denies syncope. There is no dysmenorrhea. A thyroid-stimulating test last month was within normal limits. Abdominal and pelvic exams are normal. What statement is true regarding dysfunctional uterine bleeding? Answer Choices 1 It is seen in all age groups equally 2 Dilatation and curettage is the only treatment 3 The condition is most common after 40 years of age 4 The condition is not seen in adolescents 5 It is never associated with ovulatory cycles

The condition is most common after 40 years of age Explanation Dysfunctional uterine bleeding (DUB) is a condition of irregular uterine bleeding in a patient who does not have an anatomic uterine lesion. It is most common above the age of 40 years (50% of the cases), but it is also seen in adolescents (20%), in whom it is associated with anovulatory cycles. Anovulatory cycles are characterized by abnormal levels of estrogen and may be due to estrogen withdrawal or breakthrough. A deteriorating ovarian follicular function is responsible for anovulatory bleeding during the climacteric. Other etiologies, such as polycystic ovarian disease, fibroids, and thyroid disease need to be ruled out before making the diagnosis. An endometrial biopsy or dilation and curettage (if the patient cannot undergo an endometrial biopsy in the office) can be diagnostic, but it is not curative or even therapeutic in a patient with DUB. Medical therapy, including estrogens, progestational agents, progesterone-impregnated intrauterine devices, and combination oral contraceptives are used to treat the condition.

A 20-year-old woman (G1, P0) presents for her 28th week antenatal care visit. The woman's blood group is O and is Rh-. Her husband's blood type is B and is Rh+. Her indirect Coombs test in the first antenatal care visit was negative; however, the indirect Coombs test at 28 weeks is positive. What does the mother's positive indirect Coombs test mean? Answer Choices 1 The fetus is Rh- 2 The fetus has antibodies against the mother's red cells 3 The mother has formed antibodies against Rh antigens 4 The mother has an autoimmune disorder 5 The mother is at risk for hemolysis

The mother has formed antibodies against Rh antigensExplanation The mother's positive indirect Coombs test means that she has formed antibodies against Rh antigens. The fact that the mother is Rh- and that the father is Rh+ means that the likelihood that the mother will be carrying an Rh+ baby can be either 50% or 100% depending on whether the father is heterozygous or homozygous for the Rh gene locus respectively. The antenatal screening of the mother at 28 weeks gestation was negative (showing anti-Rh antibodies), indicating her alloimmunization. The indirect Coombs test evaluates for the presence of circulating antibodies in the mother's serum and in this case showed the presence of anti-Rh antibodies in the mother's blood. Since the mother's Coombs test was negative initially and then positive at 28 weeks of her pregnancy, indicating that her child is Rh+ and has transported some of its red cells to her circulation, likely through a minor fetomaternal hemorrhage, which is most likely to occur in third trimester. Upon encounter of the Rh+ red blood cells, the mother formed anti-Rh antibodies in her blood, which were detected by the indirect Coombs test. The presence of the anti-Rh antibodies in the mother does not mean that she is at risk for hemolysis since her red cells are Rh-. The presence of an antibody in the mother's blood against a foreign antigen upon exposure to it is a normal immune response and does not represent an autoimmune disorder. The indirect Coombs test is performed on the mother and therefore can only show the presence or absence of the antibody under study (in this case anti-Rh) in the mother's blood.

A patient presents to the office at 5 weeks gestation. She has been spotting for several days, and the flow is increasing slightly. She has mild, crampy pain; no fetal heart tones are auscultated. Ultrasound reveals an intrauterine gestational sac with a fetal pole. What is the most likely diagnosis? A Threatened abortion B Spontaneous abortion C Incomplete abortion D Septic abortion

Threatened abortion A While she is at risk for a spontaneous abortion, one cannot determine fetal viability with one exam. Serial exams are necessary. Fetal heart tones are not always present at 5 weeks.

A 25-year-old gravida 1 woman who is HIV positive arrives at the hospital in early labor. Membranes are intact and the cervix is 50% effaced and 3 to 4 cm dilated. Fetal heart rate is 150 beats/min. Which of the following procedures is contraindicated during labor? A amniotomy B augmentation of labor with oxytocin C external monitoring D operative delivery E use of fetal scalp electrodes

Use of fetal scalp electrodes E Use of fetal scalp electrodes and scalp sampling is contraindicated in the HIV-positive woman because it increases the risk of vertical transmission of the human immunodeficiency virus to the infant. While ruptured membranes for more than 4 hours is associated with an increased risk of vertical transmission, amniotomy per se is not contraindicated. Augmentation of labor, external monitoring, and operative delivery are not contraindicated and, in fact, may be indicated in specific instances for the well-being of the infant and/or mother.

A 23-year-old primiparous woman with an estimated gestational age of 27 weeks presents following a motor vehicle accident. Another car struck the vehicle that she was driving on the passenger side at 40 mph. She was wearing her seat belt. She cannot recall striking her head, but her chest hurts where the belt was, and her chin is scraped from the air bag. Review of her prenatal charts shows that her laboratory work is up to date. Her Rh factor is negative and her initial antibody screen was negative. She has gained 18 pounds during the pregnancy, and her blood pressure and urine dipstick readings have been normal. Her dates were confirmed by an ultrasound done at 12 weeks, and the fundal height measurements have fit with the dates throughout the pregnancy. Her physical exam reveals a blood pressure of 130/85 mm Hg, pulse of 96 BPM, respirations of 16/minute, and temperature of 98 °F (36 °C). There is an irregular abrasion on the chin. All extremities move fully without pain. The fundal height is 28 cm and the abdomen is non-tender. The fetal head is ballotable above the symphysis. Question What would be a major warning sign of minor trauma in this patient? Answer Choices 1 Fetal heart rate of 150 beats/minute 2 Presence of short and long-term fetal heart rate variability 3 2 fetal heart rate accelerations of about 22 beats per minute within a 20 minute strip 4 2 uterine contractions per hour 5 Vaginal bleeding

Vaginal bleeding Explanation Pregnant women who are involved in motor vehicle accidents need to be evaluated for the likelihood of injury that could have adverse effects on the pregnancy. The workup should include a careful physical exam and lab testing to check for maternal or fetomaternal hemorrhage. Rhesus-negative mothers may need Rh° (D) immune globulin. The most important component of the evaluation is fetal heart monitoring of at least 4 hours duration. After this initial monitoring period, if the mother is stable and has no specified warning signs, she may be discharged. Vaginal bleeding may be indicative of placental abruption and doing a speculum examination to evaluate for the presence of amniotic fluid and/or blood is prudent. An urgent ultrasound to assess for abruption is not likely to be helpful (sensitivity is 40% and specificity is 25). Differentiating between normal 'venous lakes' and extravasation, indicating abruption, is difficult. Frequent uterine contractions with or without vaginal bleeding are a good indicator; later, a detailed biophysical profile might be helpful if initial cardiac monitoring is equivocal. In a recent related study, frequent uterine contractions of greater than 4 contractions/hr are associated with poorer outcomes (especially within the initial 4 hours after trauma). Those with fewer than 4 uterine contractions per hour had the same live birth rates and Apgar scores as the control patients. Evidence of a reactive fetal strip would include the presence of short and long-term beat-to-beat variability, and the presence of at least 2 accelerations in fetal heart rate of at least 20 beats/minute within a 20 minute strip. In addition, a fetal heart rate of between 160/minute and 120/minute (above or below which would be classed as a fetal tachycardia and fetal bradycardia, respectively). Other warning signs would include the presence of abdominal tenderness and the presence of fetal decelerations, especially late decelerations.

a 23-year-old primiparous woman with an estimated gestational age of 27 weeks presents following a motor vehicle accident. Another car struck the vehicle that she was driving on the passenger side at 40 mph. She was wearing her seat belt. She cannot recall striking her head, but her chest hurts where the belt was, and her chin is scraped from the air bag. Review of her prenatal charts shows that her laboratory work is up to date. Her Rh factor is negative and her initial antibody screen was negative. She has gained 18 pounds during the pregnancy, and her blood pressure and urine dipstick readings have been normal. Her dates were confirmed by an ultrasound done at 12 weeks, and the fundal height measurements have fit with the dates throughout the pregnancy. Her physical exam reveals a blood pressure of 130/85 mm Hg, pulse of 96 BPM, respirations of 16/minute, and temperature of 98 °F (36 °C). There is an irregular abrasion on the chin. All extremities move fully without pain. The fundal height is 28 cm and the abdomen is non-tender. The fetal head is ballotable above the symphysis. Question What would be a major warning sign of minor trauma in this patient? Answer Choices 1 Fetal heart rate of 150 beats/minute 2 Presence of short and long-term fetal heart rate variability 3 2 fetal heart rate accelerations of about 22 beats per minute within a 20 minute strip 4 2 uterine contractions per hour 5 Vaginal bleeding

Vaginal bleeding may be indicative of placental abruption and doing a speculum examination to evaluate for the presence of amniotic fluid and/or blood is prudent. An urgent ultrasound to assess for abruption is not likely to be helpful (sensitivity is 40% and specificity is 25). Differentiating between normal 'venous lakes' and extravasation, indicating abruption, is difficult. Frequent uterine contractions with or without vaginal bleeding are a good indicator; later, a detailed biophysical profile might be helpful if initial cardiac monitoring is equivocal.

A progestin-only contraceptive, or "minipill," would be most appropriate for which of the following patients? A a 25-year-old woman in excellent overall health B a 28-year-old woman with a history of epilepsy C a 32-year-old woman with a history of pelvic inflammatory disease D a 37-year-old woman who smokes 2 packs per day and has a history of hypertension E a 38-year-old woman with a history of asthma and bronchitis

a 37-year-old woman who smokes 2 packs per day and has a history of hypertension In the majority of cases, a combined hormonal contraceptive (ie, one that contains both an estrogen and progestin) is the preferred method of oral contraception because of its efficacy when used perfectly (>99%). However, for women older than 35 years of age who are smokers or are obese, or who have a history of hypertension or vascular disease, progesterone-only contraceptives are recommended. Ethinyl estradiol (EE), the most common estrogen found in combined hormonal contraceptives, has been associated with an increased risk of myocardial infarction in women older than 35 years of age who are smokers. Additionally, EE has also been shown to cause increases in blood pressure in both normotensive and mildly hypertensive women. Progestin-only contraceptives, however, tend to be less effective than the combined hormonal contraceptives.

A 37-year-old female presents to the labor and delivery department complaining of intermittent pain and contractions. Upon arrival, she also complains of vaginal bleeding. She is a G3P2 at 39 weeks gestation; no other prenatal complications are noted. She is a non-smoker. A physical exam reveals the following: P 90, BP 130/80, T 98.7°F, abdomen gravid, positive bowel sounds, and left lower quadrant tenderness noted. A sterile speculum exam reveals the cervix to be dilated 8, fetus is cephalic, and membranes are intact. The fetal monitor reveals heart tones in the 140s with mild, decreased variability and good quality contractions noted. Delivery is felt to be imminent, and vaginal delivery has been determined to be the best course of action. What will likely decrease bleeding and shorten time to delivery? A Increased activity level B Amniotomy C Oxytocin therapy D Epidural placement E IV sedation

amniotomy If the fetus is mature and vaginal delivery (versus c-section) has been determined to be the best course of action, then amniotomy may diminished amnionic fluid volume. This might also allow for better spiral artery compression, and serve to both decrease bleeding from the implantation site and reduce entry of thromboplastin into the maternal circulation.

A 28-year-old G1P0 at 37.5 weeks gestation complains of a thin, watery discharge for the last 5 hours. She has soaked 3 pads. She has no pain and the fetus continues to be active. An external fetal monitor reveals heart tones in the 140s, with variability and no contractions. The pH of the vaginal fluids is 8. What does this indicate? A Normal vaginal secretions B Amniotic fluid C Yeast vaginitis D Bacterial vaginitis E Urine

amniotic fluid B Premature rupture of membranes is the rupture of membranes before the onset of labor (within 2 hours); preterm rupture is the rupture prior to 37 weeks gestation. The absence of contraction on the monitor, in addition to no complaints indicates no labor. Urinary incontinence is common, but the ph of 8 indicates amniotic fluid.

A 22-year-old female presents to her obstetrical appointment at 39 weeks gestation. Her pregnancy to date has been uncomplicated. She is concerned that her infant may be larger than average, as her fundal height measures 41. On physical exam, her fetus is in a cephalic presentation, her cervix is soft and 1-cm dilated, and the fetus is at a -3 station. Her membranes are intact, she is not contracting, and her vitals are normal. She requests to be induced. For decreased risk of complication and optimal fetal outcome, when should she expect to be induced? A Now B At 40.5 weeks C 41.5 weeks D 42.5 weeks E Not expected

at 41.5 weeks In the absence of complication, the recommendation from ACOG is to wait for labor to occur. Large for gestational age is not an indication for induction in the absence of diabetes. Gestations greater than 42 weeks increase risk of fetal stillbirth.

You are caring for a 33-year-old G3P2 women. Her PMH is significant for obesity and allergies. She receives routine obstetrical care. When is she due for her glucose challenge testing to rule out gestational diabetes? A Between 12 and 16 weeks gestation B Between 16 and 20 weeks gestation C Between 20 and 24 weeks gestation D Between 24 and 28 weeks gestation E Between 28 and 32 weeks gestation

between 24 and 28 weeks gestation Routine surveillance for gestational diabetes of a pregnant women is between 24 and 28 weeks; it is not modified due to obesity or other risk factors for diabetes. Gestational diabetes is a hormone-mediated intolerance. Surveillance is modified in the presence of pre-existing diabetes.

A 32-year-old woman is 2-hours status post cesarean delivery of a twin gestation at 36 weeks. Her pregnancy was uncomplicated. She presented in early labor, which became prolonged despite oxytocin infusion. A cesarean section was performed when her labor became non-progressive. In the recovery area, she notes nausea and lightheadedness. On exam, her heart rate is 133 beats per minute, and blood pressure is 76/42 mm Hg. Significant vaginal bleeding is noted; abdominal palpation reveals a soft uterus. Question What intervention would be most appropriate? Answer Choices 1 Administration of intravenous magnesium 2 Bimanual uterine massage 3 Immediate surgical exploration 4 Transfusion with fresh frozen plasma 5 Transvaginal ultrasound

bimanual uterine massage Explanation The most likely cause of bleeding in this patient is uterine atony. Uterine atony occurs when the uterine myometrium fails to contract following delivery. Contractions of the uterine muscles after delivery normally tamponade bleeding from uterine arterioles. Absence of this response causes continued bleeding, which usually becomes evident early after delivery. Bimanual uterine palpation revealing a soft, 'boggy' uterus confirms the diagnosis. Initial treatment involves bimanual uterine massage, which helps promote uterine contractions. In addition, uterotonic agents are administered. The first-line of therapy is intravenous oxytocin. Second-line therapies including ergot alkaloid derivates and prostaglandins (e.g., Hemabate) are used when oxytocin therapy is unsuccessful. When these measures are unsuccessful, surgical interventions are required, the most common of which is bilateral uterine artery ligation. Uterine atony can also occur as a result of retained products of conception; these products inhibit uterine contraction. In this case, manual or surgical extraction is necessary. Intravenous magnesium is a uterine relaxant used to inhibit uterine contractions in settings of premature labor. It has the opposite effect than that desired to treat uterine atony. Transfusion with fresh frozen plasma may be indicated as a temporizing measure in the setting of disseminated intravascular coagulation accompanied by severe bleeding or massive uncontrolled hemorrhage from uterine rupture, neither of which is suspected in this clinical scenario. Transvaginal ultrasound is rarely needed to diagnose postpartum hemorrhage, and it is not the most appropriate next step in light of the patient's condition.

A previously healthy 26-year-old primiparous woman presents after 2 days of prolonged rupture of the membranes at 36 weeks of gestation with breech fetal presentation. Because of frequent fetal heart rate decelerations, the child is delivered by caesarean section. Prophylactic antibiotics were not given. Post-operatively, the woman starts complaining of increasing fatigue and muscle pain. On the 3rd day post op, she starts experiencing headache, insomnia, anxiety, generalized abdominal pain, and fever. Her temperature is 41 C. Pulse rate and respiration rate are increased (120/min and 20/min respectively). On examination, there are no signs of infection of the abdominal wall; however, there is a red rash, with swelling and tenderness, on both of her thighs. Question What is your initial step? Answer Choices 1 Antibiotics IV 2 Blood culture x 3 3 Painkillers as needed 4 Ultrasound examination 5 Antipyretics IV

blood cultures x 3 Explanation The patient most likely has puerperal sepsis. Puerperal sepsis is an infection of the genital tract that occurs at any time between the onset of the rupture of the membranes or labor and the 42nd day postpartum. Diagnostic essentials are also systemic signs of infection, abdominal and/or pelvic pain, vaginal discharge, peritoneal signs, abnormal smell/foul odor of discharge, and a delay in the rate of reduction of size of uterus (<2 cm/day during first 8 days). Collecting blood cultures prior to antibiotic administration is mandatory to identify the organism that caused sepsis. Most organisms that cause puerperal sepsis are considered normal vaginal flora, but they can cause puerperal infection in a patient with predisposing factors like: prolonged (more than 24 hours) or traumatic labor, Caesarean section, frequent and unsanitary vaginal examinations, unsanitary delivery practices, retained products of conception, hemorrhage or certain maternal conditions (anemia, malnutrition etc.), and prolonged and premature rupture of the membranes. Premature rupture of membranes happens when rupture of the membrane of the amniotic sac and chorion happens more than 1 hour before the onset of labor; it is prolonged when it occurs more than 18 hours before labor, and it is preterm when it occurs before 37 weeks gestation. This patient was at high risk for developing puerperal sepsis, and she was a good candidate for prophylactic antibiotic therapy (it was not given). You should definitively give antibiotics in the case of sepsis, but failure to check blood cultures prior to an antibiotic regimen can affect the growth of blood borne bacteria and prevent a culture from becoming positive later. You may give her painkillers, but it is essential that you deal with the cause of the sepsis. Ultrasound examination should be considered after you stabilize the patient - it is not your 1st step in the management. Antipyretics are also indicated in this patient, but blood culture is the 1st step in management of sepsis.

A hydatidiform mole is removed from the uterus of a 20-year-old woman. Subsequent to evacuation, her serum human chorionic gonadotropin (hCG) concentrations are monitored, which continue to rise. What is most likely diagnosis? Answer Choices 1 Choriocarcinoma 2 Ectopic pregnancy 3 Ovarian failure 4 Pituitary insufficiency 5 Sarcoma of the uterus

choriocarcinoma Explanation Hydatidiform mole is a form of gestational trophoblastic disease. It is a benign proliferation of trophoblastic tissue with cystic (hydropic) swelling of chorionic villi. It commonly occurs in women younger than 20 or over 45. Once a hydatidiform mole has been diagnosed, it must be removed. This can be done by either evacuating the uterus (an option preferred in younger women) or by hysterectomy. Since most trophoblastic cells secrete hCG, its levels are indicative of tumor mass, a persistently high or increasing level of hCG after removal of the mole indicates development of an invasive mole or choriocarcinoma. Hydatidiform moles are essentially due to a failed pregnancy; a second, ectopic pregnancy is extremely unlikely. Since only trophoblastic tissue produces hCG, none of the other mentioned conditions would cause an increase in its level.

The placenta has 2 portions that are identified with either the fetus or the mother. Each portion has its origin from different cells or tissues. Which of the following is the fetal portion of the placenta? Answer Choices 1 Chorion 2 Amnion 3 Yolk sac 4 Allantois 5 Decidua

chorion Explanation The fetal portion of the placenta is derived from the chorion which contains the chorionic plate and chorionic villi. The maternal portion is formed by the decidua basalis. The yolk sac develops in the first trimester to aid in the transfer of nutrients. It also has a role in blood development, is the primitive gut, and the walls contain the primordial germ cells. Usually the yolk sac detaches by the 6th week, but can persist in the adult as Meckel's diverticulum. The decidua is the endometrium found in the pregnant uterus and is shed at partition. There are several forms of decidua that occur during pregnancy and a decidual cast that can occur independent of pregnancy. The amnion can be found surrounding the fetus and amniotic fluid. It is derived from trophoblast and can be seen in the first trimester on a sonogram. Allantois is found in most mammals and serves a different function in each species. In humans its blood vessels give rise to those found in the umbilical cord. In the term fetus the allantois becomes the median umbilical ligament.

A 30-year-old G1P1 Caucasian woman with a desire to become pregnant. She has been having unprotected intercourse for 12 months without pregnancy. She typically has 3 - 4 menstrual cycles a year. She reports that she had similar problems becoming pregnant with her 1st child. Her prior doctor did a complete workup for her infertility and amenorrhea. Records have been sent for your review. She successfully conceived on a medication 3 years ago, but she cannot recall the name of the medications. She has never used any method of contraception. This patient would like you to treat her infertility. Question What is the drug of choice for this patient? Answer Choices 1 Clomiphene 2 Danazol 3 Leuprolide 4 Mifepristone 5 Spironolactone

clomiphene Explanation The drug of choice for this patient is clomiphene (Clomid and others), which has estrogen agonistic and antagonistic effects, which increase gonadotropin release and follicular maturation. This medication also improves the luteinizing hormone release and estradiol secretion. Spironolactone is the medication that inhibits aldosterone and dihydrotestosterone, functioning as an androgen receptor blocker. Its primary use in gynecology is for acne, hirsutism, and polycystic ovarian syndrome. Leuprolide is a medication that causes inhibition of gonadotropin release, which reduces ovarian steroidogenesis. It is used in women for the treatment of uterine fibroids and endometriosis; it treats prostate cancer in men. It produces the opposite desired effect when given to a patient with infertility. Mifepristone is a medication with progesterone and glucocorticoid antagonistic effects, which reduce endometrial development. It is used for early pregnancy termination; off-label, it is used for endometriosis, uterine fibroids, and postcoital contraception. This medication should not be given to this patient. Danazol is an older medication that suppresses follicle stimulating hormone (FSH) and luteinizing hormone (LH). It reduces estrogen production and is weakly androgenic. This medication would worsen the patient's chances of conception.

A 27-year-old woman presents in active labor. She is G3P2 and at 39 weeks of gestation. She has been receiving prenatal care since 6 weeks gestation, and her pregnancy has been uncomplicated. Both of her prior births were normal spontaneous vaginal deliveries. Her cervix is 6 cm, 90% effaced, mid-position, and soft. The fetus is not engaged and is thought to be vertex. Initial fetal monitoring shows a heart rate in the 140s with good accelerations, and it is reassuring. Contractions are 4 minutes apart, and she is comfortable. 20 minutes later, she experiences a large gush of clear fluid, and severe variable decelerations appear on the fetal heart rate monitor. Question What is the most likely diagnosis? Answer Choices 1 Cord prolapse 2 Placental abruption 3 Placenta previa 4 Uterine rupture 5 Vasa previa

cord prolapse Explanation The most likely diagnosis is cord prolapse. Cord prolapse occurs in 2 out of 1000 deliveries. It is diagnosed when fetal monitor recordings show that severe variable decelerations or bradycardia occur after membrane rupture. The cord is often palpable in the vagina. Cord prolapse happens most often at 5-cm cervical dilatation and in nonvertex presentations. 3 types of prolapse can occur. Overt cord prolapse is diagnosed when the membranes are ruptured and the umbilical cord falls through the cervix into the vagina ahead of the fetal presenting part. Funic presentation describes loops of umbilical cord between the presenting part and the cervical os prior to rupture of membranes. Occult cord prolapse is diagnosed when the cord is palpable alongside the presenting part on digital cervical exam. Treatment: Immediate delivery is essential to prevent fetal compromise. Cesarean delivery is generally preferred when the cervix is not fully dilated. Vacuum or forceps delivery may be attempted if the cervix is completely dilated, although manual elevation of the fetal part and emergent cesarean are the most common management. Uterine rupture is less likely since the patient has no history of cesarean delivery. Abruption is often associated with tetanic contractions and bleeding. An increase in uterine tone is seen between contractions. Vasa previa usually presents with painless bleeding at rupture of membranes. The cervical exam is not consistent with a diagnosis of placenta previa.

A 30-year-old G 2 P 1 woman comes to the emergency department at 37 weeks gestation with a chief complaint of worsening abdominal pain for the past two hours. On examination, the uterus is tense and tender. External fetal monitoring indicates frequent uterine contractions with late decelerations. An ultrasound suggests a retroplacental hemorrhage. What is the most appropriate course of action at this time? A Emergency operative delivery B Expectant management at home C Monitoring in the labor suite with the mother on her side D Tocolytic administration until the infant is 38 weeks gestation E Vaginal delivery with induction

emergency operative delivery A Indications for emergency cesarean delivery in a woman with placental abruption include fetal heart tracings that are not reassuring. Furthermore, a retroplacental hemorrhage carries a direr prognosis than a subchorionic one. Expectant management (B), monitoring (C), attempting to stop labor (D), and attempting a vaginal delivery (E) put both mother and fetus at severe risk of death.

A 27-year-old G1 P1 woman has recently given birth by caesarean section to a 36-week male infant. She did well throughout her pregnancy up until 34 weeks gestation. At that time, she presented with fever, abdominal pain, and wetness. She was diagnosed with preterm premature rupture of membranes (PPROM) and chorioamnionitis. She was treated with steroids and antibiotics, stabilized, and she then delivered by C-section with no complications. On postpartum day 7, she presents with sore breasts from breast-feeding and a sore abdomen. She also admits to an odorous vaginal discharge, but she denies any associated vaginal bleeding. On physical examination, she is having a moderate amount of lochia alba, and her temperature is 101.2°F. Question What is the most likely diagnosis at this time? Answer Choices 1 Deep vein thrombosis (DVT) 2 Endometritis 3 Infected retained placenta 4 Mastitis 5 Septic pelvic thrombophlebitis

endometritis Explanation Postpartum endometritis is most likely in this patient given her history of chorioamnionitis. Prolonged rupture of membranes, cesarean delivery, prolonged labor, and multiple cervical examinations are all risk factors for postpartum endometritis. The presence of intra-amniotic infection increases the risk of postpartum endometritis further. Antibiotics are not routinely continued for chorioamnionitis after a delivery because the "source" of the infection (the placenta) has been removed. Whenever fever occurs in the immediate postpartum period, endometritis should be suspected. Broad-spectrum antibiotics should be administered promptly by the parenteral route. Similar to chorioamnionitis, multiple bacterial organisms (usually normal vaginal and perineal flora) are likely to be responsible for this infection. Therefore, uterine cultures are unlikely to be helpful for guiding antibiotic therapy. Parenteral therapy should be continued until the patient has been afebrile for at least 24 hours. Mastitis is characterized by a swollen, firm, tender breast with systemic symptoms including inflamed breast, fevers, chills, and flu-like symptoms. Staphylococcus aureus is the typical pathogen. However, in the immediate postpartum period, breast engorgement without infection is the most likely reason for the patient's sore breasts. Pregnancy and the postpartum period increase a woman's risk of thrombogenesis. However, DVT is not a likely source of the fever. Septic pelvic thrombophlebitis is a diagnosis of exclusion and is usually entertained when fever spikes continue following treatment for endometritis. Infected retained placenta is unlikely in the absence of vaginal bleeding. Lochia This is the uterine discharge following delivery and lasts for 3 or 4 weeks. Foul-smelling lochia suggests infection. Types: (1) Lochia rubra. This blood-stained fluid lasts for the first few days. (2) Lochia serosa. This discharge appears 3 to 4 days after delivery. It is paler than lochia rubra because it is admixed with serum. (3) Lochia alba. After the 10th day, because of an admixture with leukocytes, the lochia assumes a white or yellow-white color. Puerperium: This period of 4 to 6 weeks starts immediately after delivery and ends when the reproductive tract has returned to its non-pregnant condition. Multiple anatomic and physiologic changes occur during this time; the potential exists for significant complications, such as infection or hemorrhage. Puerperal infection is defined as any infection of the genitourinary tract during the Puerperium accompanied by a temperature of 100.4oF (38 oC) or higher that occurs for at least 2 of the first 10 days postpartum, exclusive of the first 24 hours. Prolonged rupture of the membranes accompanied by multiple vaginal examinations during labor is a major predisposing cause of puerperal infection.

A 23-year-old Hispanic female delivers a healthy male child by normal vaginal delivery. She had a small episiotomy which was clean cut. She is discharged home after 48 hours but returns to the ER on the fifth post partum day with fever and chills. She says she has had foul-smelling vaginal discharge and lower abdominal pain for the last 24 hours. On exam, she has a temperature of 102° F, BP 110/60 mm of Hg, pulse 118/min, and SPO2 92%. Lungs are clear, and she has no pallor, cyanosis, or icterus. Abdominal exam demonstrates tenderness in the suprapubic area and both lower quadrants. Pelvic exam shows foul smelling purulent vaginal discharge and tender uterus. The episiotomy site is non-tender. Labs show Hb 0f 11g/dl, WBC 13,500, bands 11%, and platelets 350,000/uL. Urinalysis is pending. Question The most likely diagnosis is: Answer Choices 1 Endometritis 2 Pyelonephritis 3 Wound cellulitis 4 Cervicitis 5 Cystitis

endometritis Explanation This female patient is suffering from post partum endometritis as obvious by recent delivery, fever, foul-smelling vaginal discharge, and tender pelvic exam. Other risk factors for endometritis include invasive gynecological procedures, retained products of conception, intrauterine devices, submucosal fibroids, multiple sexual partners, unprotected intercourse with infected partner who may be asymptomatic, etc. Leukocytosis with bandemia supports diagnosis. Blood cultures, urine culture, and cervical culture may be done but treatment should not be delayed. Broad spectrum antibiotics to cover beta lactamase and anaerobic organisms like clindamycin with an aminoglycoside are given. Pyelonephritis is unlikely since she has no back pain or costovertebral angle tenderness, and foul vaginal discharge is absent in pyelonephritis. UA and urine culture should rule it out. Wound infections are rare in a small, clean-cut episiotomy. Fever with foul vaginal discharge does not occur, and the wound area will be obviously erythematous, indurated, and tender. Even in third or fourth degree lacerations antibiotic therapy is usually unnecessary. Cervicitis is either asymptomatic or has purulent vaginal discharge without fever, chills, tender abdominal exam, or leukocytosis. Cystitis may be associated with low grade fever and suprapubic tenderness, but dysuria is predominant and vaginal discharge absent. UA confirms diagnosis.

For the past year, a 30-year-old woman and her husband have been trying unsuccessfully to become pregnant. Over-the-counter ovulation tests have indicated that she is ovulating. Neither partner smokes cigarettes, uses any mind-altering drugs, or has a history of sexually transmitted infection. She has no history of abdominal surgery or pelvic procedures, diethylstilbestrol (DES) exposure, or major illness. On examination, she is 66" tall, weighs 135#, and appears healthy. Her thyroid is nonpalpable, and pelvic examination is unremarkable. What is the most appropriate next step in evaluation of this couple's infertility? A Endometrial biopsy B Examination of the husband C Pelvic ultrasonography D Performing a hysterosalpingogram E Referral to an infertility clinic

examination of the husband Male factors account for 25-40% of infertility. Since the woman's initial evaluation appears normal and ovulation tests are positive, the next step is to evaluate the husband before embarking on a more detailed evaluation of the wife. An endometrial biopsy (A) may be appropriate during the late luteal phase to evaluate the endometrial lining. Pelvic ultrasonography (C) may help identify ovarian cysts, endometrial implants, leiomyomas and other treatable conditions. Hysterosalpingography (D) is useful in evaluating the patency of the fallopian tubes and the size and shape of the uterine cavity. Since this woman is only 30 years of age and many other studies can be performed in ob-gyn setting, referral to an infertility clinic (E) is premature.

A 19-year-old G1P0 presents for her routine obstetric exam. She is at 34 weeks gestation. When completing the physical assessment you perform a Leopold maneuver. What does this maneuver assess? A Fetal lie and station B Fetal position and presentation C Fetal lie and position D Fetal lie and presentation E Fetal position and station

fetal lie and position C The maneuver is used to examine the abdomen and determine the lie (first) and position (second). While the abdominal exam can be used to determine breech versus cephalic, it does not differentiate between, face, brow, and footling presentations. A vaginal exam must be done to determine the presenting part and decent into the pelvis.

A 23-year-old primigravida is admitted to your ward because of vaginal bleeding and cramping at 26 weeks. The ultrasound shows fetal heartbeat, and you try to stop a miscarriage by ordering terbutaline. What do you tell the nurse to watch for? Answer Choices 1 Constipation and dry mouth 2 Headache, tachycardia, and nausea 3 Sedation, urinary retention, and dry mouth 4 Anxiety, headache, and mydriasis 5 Cold extremities, arrhythmia, and insomnia

headache, tachycardia and nausea Explanation Terbutaline relaxes smooth muscle in the bronchial system and the uterus by stimulating β2-receptors. It is a tocolytic drug, and most commonly used in patients with bronchospasms caused by reversible obstructive airway disease. Headache, tachycardia, and nausea are common side effects of beta-mimetic drugs, which relax the uterine muscle. Other side effects are shown in the following table. Central nervous system Drowsiness, dizziness, headache, tremor, nervousness Gastrointestinal Nausea, vomiting Cardiovascular Palpitations, tachycardia, arrhythmia Respiratory Paradox bronchospasm Metabolic Hypokalemia Skin Diaphoresis Other Dry mouth and throat Constipation and dry mouth are side effects of skeletal muscle relaxants, like cyclobenzaprine, which is used as short-term treatment of muscle spasms. Sedation, urinary retention, and dry mouth are side effects of antihistamine drugs, which effect the peripheral H1-receptors. Urinary retention and sedation however are side effects of the "older" drugs like brompheniramine, chlorpheniramine, clemastine, promethazine, and triprolidine and not the "newer" ones like fexofenadine, loratadine, and cetirizine. Anxiety, headache, and mydriasis are signs of atropine overdosage. Atropin is an anticholinergic that inhibits acetylcholine at the parasympathica neuroeffector junction. It blocks vagal effects on the sinuatrial and atrioventricular nodes thereby enhancing conduction through the AV node and increasing the heart rate. It is used to treat bradycardia and to dilate the pupils. Cold extremities, bronchospasm, and insomnia are side effects of beta-blockers. Beta-blockers (or symppatholytics) decrease myocardial contractility, heart rate, blood pressure, and cardiac output (less blood getting to the periphery, which explains cold extremities). Myocardial oxygen use gets reduced. Since it decreases the effect of catecholamines on the bronchial system, it can trigger an asthma attack by increasing contraction of bronchioli and alveoli.

A mother brings in her 20-month-old female child to the office because she noticed pubic hair growing. On examination, the clinician notices that the clitoris is enlarged; the rest is unremarkable. Which of the following is an expected laboratory finding on this patient? A increased aldosterone B increased estrogen C increased androstenedione D increased luteinizing hormone

increased androstenedione C Infant girls presenting with signs of precocious puberty need to be screened for congenital adrenal hyperplasia (CAH). CAH most commonly presents with pseudohermaphroditism in females—urogenital sinus, enlarged clitoris, or other signs of virilization. In males, there tends to be isosexual precocity in older males and salt-losing crisis in infant males. Both children show increased linear growth and skeletal maturation. The most common type of CAH is a deficiency in the enzyme 21-hydroxylase and laboratory tests demonstrate increased urinary and plasma androgens (DHEA, androstenedione). There may be elevated progesterone, but typically there is no effect on estrogen. There is also decreased aldosterone and elevated urinary ketosteroids. There is also no effect on the levels of leuteinizing hormone or follicle-stimulating hormone. Treatment usually involves glucocorticoids, mineralocorticoids, and reconstructive surgery, if needed.

A 21-year-old female presents to clinic complaining of mild, low abdominal ache and intermittent dysuria. She denies N/V/D, and she is sexually active and uses condoms some of the time. Her LMP was 10 days ago, and she is a G0P0. Physical exam reveals a healthy female in no acute distress. Vitals are as follows: P 70, BP 120/80, T 99.9°F. Lungs are clear, CV RRR, abd soft non-tender, + BS. Pelvic exam reveals normal external genitalia, scant discharge, moderate cervical motion tenderness, and no adnexal masses. What is her most likely diagnosis? A Tubo-ovarian abscess B Gastroenteritis C Ectopic pregnancy D Cervicitis E Pelvic inflammatory disease

pelvic inflammatory disease Suspicion for PID should be very high in a young, healthy, and sexually active woman with cervical motion tenderness. She is not spotting and just menstruated, making ectopic pregnancy much less likely.

A 30-year-old patient presents to labor and delivery complaining of bright red vaginal bleeding. She has no pain. The fetus is still active. She is 37 weeks pregnant. PMH is significant for in vitro fertilization. What is the most likely diagnosis? A Placental abruption B Placenta acreata C Placenta previa D Disseminated intravascular coagulopathy E Active labor

placenta previa C The most likely diagnosis is placenta previa, as the bleeding is bright red and painless. Labor and abruption are associated with discomfort and pain.

A 29-year-old woman, G4P2011, LMP 9 months prior, presents with severe lower abdominal pain. The pain is sharp and tearing. She began to have vaginal spotting prior to presentation. She denies any contractions. There is no history of medical problems or surgery, and she is on no medications. All previous deliveries were vaginal. She has smoked 1 pack of cigarettes a day over the past 10 years, denies alcohol use, but does admit to a remote history of heroin abuse by insufflation. On physical examination: T = 100.4° F; BP= 110/70 mmHg; P= 85/min; RR= 20/min. Pertinent findings on the PE were relegated to the pelvic exam; fundal height measures 39 cm, and there is profuse bleeding from the vagina. Fetal monitor shows contractions every minute with elevated baseline uterine tone. Fetal tachycardia is evident at 180 beats/minute, and late decelerations are also present. Question What is the most likely diagnosis? Answer Choices 1 Cervical cancer 2 Chorioamnionitis 3 Placenta previa 4 Placental abruption 5 Uterine rupture

placental abruption Explanation This patient has placental abruption. Abruption is a leading cause of 2nd and 3rd trimester bleeding. A meta-analysis has demonstrated that smoking increases the risk of abruption by 90%. Maternal and paternal smoking increases the risk of abruption 2-fold. Risk increases 5-fold when both parents smoke. Women with a history of abruption have a 15% increased risk during future pregnancies. Placenta previa, cocaine use, preeclampsia, and preterm premature rupture of membranes (PPROM) are also associated with placental abruption. Placental abruption is defined as complete or partial separation of the placenta prior to delivery. The incidence of abruption is 5 or 6 out of 1000 deliveries. Obvious vaginal bleeding occurs if the hemorrhage develops between the membranes and the uterus. Concealed presentations occur when blood collects behind the placenta. Abruption is a leading cause of 2nd and 3rd trimester bleeding, and it causes significant maternal and neonatal morbidity and mortality. Classically placental abruption presents as painful third-trimester bleeding. Treatment:Emergent cesarean is generally indicated when the patient is not in labor and hemodynamic compromise is present. If delivery is imminent and the abruption is mild, vaginal delivery may be attempted. Cervical cancer is incorrect.Although cervical cancer can coexist with pregnancy, it is very unlikely in this case. The severity of the signs and symptoms in this case would be incongruent with achieving 9 months gestation. Chorioamnionitis is incorrect.Clinical features include maternal fever and uterine tenderness in the presence of confirmed premature rupture of membrane (PROM). Profuse vaginal painful bleeding is not a finding. Placenta previa is incorrect, as it classically presents with painless 3rd trimester bleeding. Uterine rupture is incorrect. A uterine rupture typically occurs during active labor. As this patient denies any contractions, it is very unlikely.

You are providing care for a 28-year-old who presents to clinic for her 27-week obstetrical check. She has no complaints and the fetus is active. Physical exam reveals the following: P 88, BP 142/90, FHT 148, UA negative. CBC is normal. Her BP pre-partum was 110/70. What is the most likely diagnosis at this point? A Pregnancy induced hypertension B Preeclampsia C Eclampsia D Chronic hypertension E stress

pregnancy induced htn HTN is one of the triad of symptoms of preeclampsia, but at this point there are no other symptoms or proteinuria. She will need to be carefully monitored, as 50% of these presentations will progress to preeclampsia.

You are providing care for a 21-year-old G1P1 who delivered a 3990 gram infant by normal spontaneous vaginal delivery. After delivery, she continues to bleed vaginally, more than is expected for routine delivery. What is the most likely cause of her hemorrhage? A cervical laceration B lateral vaginal wall laceration C retained placenta D uterine atony E placenta acreata

uterine atony D Uterine atony continues to be the most common cause of postpartum hemorrhage, even though all of the answer choices may cause it.

A 32-year-old woman is 2-hours status post cesarean delivery of a twin gestation at 36 weeks. Her pregnancy was uncomplicated. She presented in early labor, which became prolonged despite oxytocin infusion. A cesarean section was performed when her labor became non-progressive. In the recovery area, she begins to complain of nausea and lightheadedness. On exam, her heart rate is 133 beats per minute and blood pressure is 76/42. Significant vaginal bleeding is noted, and abdominal palpation reveals a soft uterus. Question What is the most likely diagnosis? Answer Choices 1 Cervical laceration 2 Placenta accreta 3 Prolapsed uterus 4 Uterine atony 5 Uterine rupture

uterine atony Explanation The most likely cause of bleeding in this patient is uterine atony. Uterine atony is when the uterine myometrium fails to contract following delivery. Contractions of the uterine wall after delivery normally tamponade bleeding by tightening around uterine arterioles. Absence of this response causes continued bleeding, which usually becomes evident early after delivery. Bimanual uterine palpation revealing a soft, "boggy" uterus is what typically confirms the diagnosis. Risk factors include uterine distension caused by multiple gestations or polyhydramnios, as well as prolonged oxytocin use prior to delivery, placental previa, and chorioamnionitis. Treatment involves uterine massage and the use of medications that promote uterine contractions, including oxytocin, ergot alkaloids, and prostaglandins (e.g., Hemabate). Uterine atony can also occur as a result of retained products of conception; these products inhibit uterine contraction. In this case, manual or surgical extraction is necessary. Lower genitourinary tract or rectal tears may cause bleeding after delivery, the site of which is often visible on pelvic exam and amenable to suture ligation. Neither of these conditions impairs uterine contractions. Placenta accreta describes a condition in which the placenta implants within the uterine myometrium. Although it can be a cause of postpartum hemorrhage, it is relatively less common than uterine atony and would have been recognized at the time of cesarean delivery. Uterine prolapse (also called uterine inversion) describes the collapse of the uterine fundus through the cervical os. The uterus may be seen protruding from the vagina and abdominal exam reveals absence of the uterine body. It is a rare but serious event requiring rapid correction to restore hemodynamic stability. Uterine ruptureis also a rare complication involving a complete tear of the uterine layers with extrusion of uterine content into the abdominal cavity. It most commonly occurs at the site of a uterine scar from prior cesarean section; and it often happens before delivery of the fetus, requiring emergent laparotomy.


Ensembles d'études connexes

BIO 1520 Module 2 Learning Catalytics

View Set

Chapter 6: Skeletal System: Bones and Bone Tissue

View Set

Chapter 6: Adolescence: Cognitive and Social Development

View Set

ACC 301 Ch.2 Smartbook Questions

View Set

Leadership, Delegation, Priority client care

View Set

Chapter 10- Performance Nutrition

View Set